cart-icon Товаров: 0 Сумма: 0 руб.
г. Нижний Тагил
ул. Карла Маркса, 44
8 (902) 500-55-04

Примеры по математике на умножение и деление 5 класс: Умножение и деление натуральных чисел – примеры (5 класс, математика)

Содержание

Самостоятельные работы по математике за 5 класс к учебнику Виленкина Н.Я. за 1, 2, 3 и 4 четверти

Дата публикации: .

Самостоятельные на темы: «Натуральные числа и их обозначения», «Сложение и вычитание натуральных чисел», «Сравнение натуральных чисел», «Отрезок, прямая, луч», «Умножение натуральных чисел», «Деление натуральных чисел», «Выражения и уравнения», «Квадрат и куб числа», «Окружность и круг», «Обыкновенные дроби», «Сравнение дробей» и др.

Некоторые понятия к учебному материалу.

1. Натуральные числа – используются для счета предметов в повседневной жизни.
2. Отрезок. Длина отрезка – расстояние между его крайними точками, концами. Обозначается заглавными латинскими буквами, например AB.
3. Шкала – специальная линейка с делениями (штрихами).
4. Единичный отрезок – отрезок с длиной равной единице.
5. Меньше и больше. Меньше, число, которое при счете называется раньше. Больше, число, которое при счете называется позже.
6. Слагаемые числа – числа, которые складываются.
7. Вычитание. Число из которого вычитают – это уменьшаемое. Число, которое вычитается – это вычитаемое. В итоге получаем разность.

Самостоятельная работа №1 (входная работа на повторение)


Вариант I.

1. Определение числа.

а) Определите натуральное число, которое следует за числом 699.
б) Определите натуральное число, которое на две единицы меньше числа 1001.
в) Определите натуральное число, которое на единицу больше числа 239 999.
г) Определите натуральное число, которое на единицу меньше числа 394 000.

2. Решите задачу.

В городском сквере посажено 340 деревьев. А в парке посажено 270 деревьев. На сколько деревьев больше в городском сквере, чем в парке?

3. Решите примеры.

а) 492 + 1 220 =б) 3 495 — 593 =
в) 5112 : 6 =г) 56 * 23 =

Вариант II.

1. Определение числа.

а) Определите натуральное число, которое следует за числом 879.
б) Определите натуральное число, которое на единицу меньше числа 2 000.
в) Определите натуральное число, которое на единицу больше числа 722 999.
г) Определите натуральное число, которое на единицу меньше числа 24 000.

2. Решите задачу.

Рыбаки за первую неделю поймали 395 кг рыбы, а за вторую неделю – 239 кг. На сколько кг было поймано меньше за вторую неделю, чем за первую?

3. Решите примеры.

а) 638 + 1 445 =б) 6 112 — 2 598 =
в) 2688 : 3 =г) 24 * 45 =

Вариант III.

1. Определение числа.

а) Определите натуральное число, которое следует перед числом 699.
б) Определите натуральное число, которое на единицу меньше числа 3 000.
в) Определите натуральное число, которое на единицу больше числа 28 999.
г) Определите натуральное число, которое на единицу меньше числа 12 000.

2. Решите задачу.

В огороде посадили 2 грядки помидор. С первой грядки было собрано 427 помидор, а со второй грядки – 311. На сколько меньше помидор было собрано со второй грядки, чем с первой?

3. Решите примеры.

а) 455 + 3 412=б) 5 332 — 593 =
в) 3648 : 8 =г) 29 * 41 =

Самостоятельная работа №2 на тему: «Натуральные числа и их обозначения»


Вариант I.

1. Запишите следующие числа 3 раза подряд и запишите полученное в результате число в виде словосочетания.

а) число 20;
б) число 49.

2. Представьте следующие словосочетания в числовом виде.

а) Шесть миллиардов пятьсот три тысяча семь.
б) На единицу больше чем пятьсот девять тысяч девятьсот девяносто девять.

3. Определите все возможные трехзначные числа, состоящие из следующих чисел (числа не должны повторяться).

a) 2, 3 и 7.
b) 4, 0 и 9.

Вариант II.

1. Запишите следующие числа 3 раза подряд и запишите полученное в результате число в виде словосочетания.

а) число 60;
б) число 38.

2. Представьте следующие словосочетания в числовом виде.

а) Восемь миллиардов триста одна тысяча три.
б) На единицу больше чем сто девять тысяч девятьсот девяносто девять.

3. Определите все возможные трехзначные числа, состоящие из следующих чисел (числа не должны повторяться).

a) 1, 3 и 9.
b) 2, 4 и 0.

Вариант III.

1. Запишите следующие числа 3 раза подряд и запишите полученное в результате число в виде словосочетания.

а) число 30;
б) число 58.

2. Представьте следующие словосочетания в числовом виде.

а) Два миллиарда шестьсот два миллиона триста.
б) На единицу больше чем семьсот пять тысяч девятьсот девяносто восемь.

3. Определите все возможные трехзначные числа, состоящие из следующих чисел (числа не должны повторяться).

a) 5, 2 и 8.
b) 1, 3 и 0.

Самостоятельная работа №3


Вариант I.

1. Переведите из одной единицы измерения в другую.

а) 8 дм 43 см = … смб) 5 км 549 м = … м
в) 7 см 18 мм = … ммг) 249 см =… дм … см

2. Начертите отрезок AB, равный 17 см 5 мм. Отметьте на нем точки C и D. AC равно 10 см 4 мм, CD равно 4 см 9 мм. Чему равна длина отрезка DB?

3. Решите задачу.

Перед домом построили забор. Забор держится на 18 столбах, расстояние между столбами составляет пять метров. Каково расстояние между шестым и четырнадцатым столбами?

4. Начертите четырехугольник ABCD. Отметьте точкой T середину стороны BC. Соедините точки B и D, А и T. Выпишите все многоугольники, которые образовались.


Вариант II.

1. Переведите из одной единицы измерения в другую.

а) 4 дм 23 см = … смб) 25 км 50 м = … м
в) 16 см 65 мм = … ммг) 456 см =… дм … см

2. Начертите отрезок AB, равный 15 см 4 мм, отметьте на нем точки C и D. AC равен 8 см 2 мм, CD равен 3 см 7 мм. Чему равна длина отрезка DB?

3. Решите задачу.

Перед домом построили забор. Забор держится на 19 столбах, расстояние между столбами составляет 4 метра. Каково расстояние между третьим и восьмым столбами?

4. Начертите четырехугольник ABCD. Отметьте середину AB и поставьте точку N. Проведите отрезки DN и АС. Выпишите все многоугольники, которые образовались.


Вариант III.

1. Переведите из одной единицы измерения в другую.

а) 19 дм 5 см = … смб) 21 км 678 м = … м
в) 43 см 8 мм = … ммг) 503 см =… дм … см

2. Начертите отрезок AB, равный 13 см 2 мм, отметьте на нем точки C и D. AC равен 7 см 3 мм. CD равен 3 см 6 мм. Чему равна длина отрезка DB?

3. Решите задачу.

Перед домом построили забор. Забор держится на 16 столбах, расстояние между столбами составляет 3 метра. Каково расстояние между пятым и одиннадцатым столбами?

4. Начертите четырехугольник ABCD. Отметьте середину CD и поставьте точку М. Проведите отрезки BM и АС. Выпишите все многоугольники, которые образовались.

Самостоятельная работа №4 на тему: «Сравнение натуральных чисел»


Вариант I.

1. Сравните числа.

а) 3 485 660 … 3 458 660б) 303 559 … 330 559
в) 2 596 440 … 2 569 440г) 45 696 … 44 696

2. Представьте в виде двойного неравенства: 18 т 347 кг … 18 т 4 ц 59 кг … 18 568 кг.


Вариант II.

1. Сравните числа.

а) 34 686 887 … 34 868 887б) 3 855 … 3 585
в) 40 955 999 … 40 595 999г) 455 776 … 445 776

2. Представьте в виде двойного неравенства: 13 км 845 м … 14675 м … 13 км 845 м 3 дм.


Вариант III.

1. Сравните числа.

а) 678 881 … 687 881б) 782 223 … 728 223
в) 2 490 606 … 2 490 660г) 13 799 … 13 977

2. Представьте в виде двойного неравенства: 15 т 475 кг … 15657 кг … 157 ц 35 кг.


Самостоятельная работа №5 на тему: «Сложение и вычитание натуральных чисел»


Вариант I.

1. Выполните сложение.

а) 348 588 667 + 239 586 394 = б) 93 955 483 + 495 868 991 =
в) 23 394 596 + 5 697 345 =г) 3 949 532 + 405 669 =

2. Выполните вычитание.

а) 348 588 667 — 283 745 733 =б) 93 955 483 — 22 394 583 =
в) 23 394 596 — 192 485 =г) 3 949 532 — 4 348 483 =

3. Решите задачу.

Мастерская закупила 560 гаек. На ремонт первой машины потребовалось 203 гайки, а на ремонт второй машины – еще 293 гайки. Сколько гаек осталось в мастерской?

4. Решите задачу.

В концертном зале стояло 454 стула. Для проведения концерта принесли 123 новых стула, а после антракта – еще 13 стульев. Сколько всего стульев стало в концертном зале?

Вариант II.

1. Выполните сложение.

а) 3 484 558 + 9 499 834 =б) 93 955 483 + 394 585 665 =
в) 3 495 863 + 35 384 588 =г) 5 697 291 + 34 405 669 =

2. Выполните вычитание.

а) 4 856 342 — 3 495 384 =б) 283 495 864 — 232 485 965 =
в) 5 965 493 — 3 449 594 =г) 23 455 303 — 19 485 588 =

3. Решите задачу.

В рулоне было смотано 327 м ленты. В первый день использовали 103 м, а во второй день – ещё 205 м. Сколько метров осталось в рулоне?

4. Решите задачу.

В магазине находилось 4 т 150 кг сахара. В первый день привезли 340 кг сахара, а во второй день – еще 4 ц сахара. Сколько кг сахара стало в магазине?

Вариант III.

1. Выполните сложение.

а) 2 399 388 + 239 586 394 =б) 435 483 + 495 868 991 =
в) 34 567 784 + 13 412 345 =г) 6 563 544 + 23 876 554 =

2. Выполните вычитание.

а) 455 586 661 — 283 745 733 =б) 40 954 586 — 22 394 583 =
в) 495 568 222 — 448 568 338 =г) 3 949 532 — 2 349 588 =

3. Решите задачу.

В моток смотано 459 м провода. В первый день истратили 119 м, а на второй день – 239 м провода. Сколько метров провода осталось в мотке?

4. Решите задачу.

На складе находилось 3 т и 450 кг муки. В первый день привезли 560 кг, через неделю привезли еще 5 ц муки. Сколько кг муки стало на складе?

Самостоятельная работа №6


Вариант I.

1. Найдите значение выражения: ( а + 46 ) : ( b — 48 ), если а = 35 и b = 57.

2. Упростите выражения.

а) с + 239 — 93;
б) 485 — 483 + d.

3. Составьте уравнение для решения задачи и решите его.

Было задумано некоторое число. К нему прибавили число 194, а потом прибавили ещё число 110 и получили число 322. Какое число было задумано?

4. Решите уравнения.

a) (305 — ( ( 45 + х ) — 32 ) + 96 = 223;
б) 38 + ( 69 — y ) + 74 = 172.

Вариант II.

1. Найдите значение выражения: ( а — 34 ) * ( b + 9 ), если а = 60 и b = 11.

2. Упростите выражения.

а) 594 — 69 — а;
б) 149 + b — 54.

3. Составьте уравнение для решения задачи и решите его.

Было задумано некоторое число. Из этого числа вычли число 424, а затем прибавили число 392. В итоге, получилось число 632. Какое число было задумано?

4. Решите уравнения.

a) 209 — ( ( 145 + х ) — 12 ) + 96 = 123;
б) 18 + ( 159 — y ) + 34 = 172.

Вариант III.

1. Найдите значение выражения: ( а — 68 ) : b + 2 339, если а = 92 и b = 8.

2. Упростите выражения.

а) с + 239 — 193;
б) 485 — d + 384.

3. Составьте уравнение для решения задачи и решите его.

Было задумано некоторое число. Из этого числа вычли число 209, а затем прибавили число 47. В итоге, получилось число 217. Какое число было задумано?

4. Решите уравнения.

a) ( 111 — ( 45 + х ) ) + 96 = 123;
б) 29 + ( 59 — y ) + 15 = 72.

После завершения второй четверти, учащиеся должны:
1. уметь умножать натуральные числа и использовать эти знания;
2. уметь производить деление натуральных чисел, в том числе и деление с остатком, и использовать эти навыки при решении задач;
3. знать распределительное свойство умножения, уметь применять это свойство при устных вычислениях и при решении задач;
4. знать, что такое возведение числа в степень. Понимать, что такое корень и куб числа;
5. понимать, что такое формула, и как производить вычисления по формуле.

Самостоятельная работа №7 на тему: «Действия с натуральными числами. Умножение»


Вариант I.

1. Выполните умножение.

а) 283 * 46 =б) 29 * 473 =в) 841 * 93 =г) 19 * 632 =
д) 570 * 340 =е) 930 * 730 =ж) 5100 * 360 =з) 560 * 230 =

2. Умножьте числа, используя наиболее удобный порядок действий.

а) 25 * 491 * 4 * 200 =
б) 4 * 324 * 25 * 300 =

3. Расположите уравнения в порядке убывания, не производя никаких действий.

35 * 34 =34 * 33 =34 * 36 =32 * 32 =

4. Решите задачу.

В двухэтажной школе всего 32 кабинета и в каждом кабинете по 12 парт. В трехэтажной школе 45 кабинетов и в каждом кабинете по 14 парт. Сколько всего парт необходимо городским школам, если в городе 8 двухэтажных и 5 трехэтажных школ?

Вариант II.

1. Выполните умножение.

а) 342 * 57 =б) 64 * 268 =в) 342 * 89 =г) 32 * 864 =
д) 920 * 560 =е) 470 * 990 =ж) 2300 * 630 =з) 430 * 540 =

2. Умножьте числа, используя наиболее удобный порядок действий.

а) 25 * 376 * 4 * 500 =
б) 4 * 265 * 25 * 200 =

3. Расположите уравнения в порядке убывания, не производя никаких действий.

85 * 84 =84 * 83 =84 * 86 =82 * 82 =

4. Решите задачу.

В поселке построено 18 домов. Из них 4 трехэтажных, 6 двухэтажных, остальные одноэтажные дома. В трехэтажных домах – 18 окон, в двухэтажных – 14 окон, в одноэтажных – 8 окон. Сколько окон необходимо для 4 таких же посёлков?

Вариант III.

1. Выполните умножение.

а) 563 * 24 =б) 32 * 441 =в) 324 * 87 =г) 23 * 728 =
д) 220 * 680 =е) 240 * 580 =ж) 7500 * 290 =з) 920 * 630 =

2. Умножьте числа, используя наиболее удобный порядок действий.

а) 25 * 376 * 4 * 300 =
б) 4 * 641 * 25 * 100 =

3. Расположите уравнения в порядке убывания, не производя никаких действий.

65 * 64 =64 * 63 =64 * 66 =62 * 62 =

4. Решите задачу.

В один мешок помещается 26 кг картофеля, или 34 кг муки, или 38 кг сахара. Сколько всего весит груз, если в машину погрузили 32 мешка картофеля, 38 мешков муки и 52 мешка сахара?

Самостоятельная работа №8 на тему: «Деление натуральных чисел»


Вариант I.

1. Выполните деление.

а) 475 860 : 5 =б) 8 412 : 4 =в) 492 000 000 : 1 000 =
г) 270 930 : 3 =д) 386 240 : 5 =е) 19 688 : 23 =

2. Решите уравнения.

а) X : 85 = 2 210б) 36 690 : Y = 10в) 792 : X = 4
г) 15 * ( 39 : X ) = 45д) Y : 42 = 168е) 65 065 : Y = 1 001

3. Решите задачу.

Фермеру необходимо вспахать поле размером 318500 м. За сколько дней он вспашет поле, если известно, что за день он может вспахать 45 500 м?

4. Остаток равен 18, неполное частное – 35 и делитель – 23. Найдите делимое.


Вариант II.

1. Выполните деление.

а) 489 560 : 5 =б) 36 690 : 3 =в) 657 000 : 1 000 =
г) 960 552 : 6 =д) 522 240 : 2 =е) 67 065 : 85 =

2. Решите уравнения.

а) X : 26 = 456б) 4 760 : Y = 85в) 792 : X = 8
г) 35 * ( 54 : X ) = 315д) Y : 3 = 3015е) 524 : Y = 131

3. Решите задачу.

Станок производит 1200 заготовок за 1 час. Сколько минут нужно машине, чтобы приготовить 48 000 заготовок?

4. Остаток равен 33, неполное частное – 41 и делитель – 25. Найдите делимое.


Вариант III.

1. Выполните деление.

а) 236 560 : 4 =б) 36 690 : 6 =в) 612 345 000 : 1 000 =
г) 960 440 : 8 =д) 678 350 : 2 =е) 31 464 : 69 =

2. Решите уравнения.

а) X : 25 = 14б) 1 820 : Y = 28в) 1 836 : X = 6
г) 52 * Y = 468д) Y : 3 = 7 659е) 1048 : Y = 131

3. Решите задачу.

Комбайн убирает 30 га пшеницы за 1 час. Сколько дней ему нужно, чтобы убрать площадь равную 1200 га, если в день он будет работать по 10 часов?

4. Остаток равен 24, неполное частное – 25 и делитель – 28. Найдите делимое.


Самостоятельная работа №9 на темы: «Выражения, уравнения и решение уравнений», «Квадрат и куб числа»


Вариант I.

1. Решите примеры.

а) 34 + ( 239 — 606 : 6 ) * 4 — 393 : 3 =
б) 152 =
в) 73 =
г) ( 14 + 7 )2 — ( 5 + 13 )2 + 287 =

2. Упростите выражение и найдите его значение при с=34: 47с + 34 — 58 + 12с — 58.

3. Решите уравнения.

а) 15 * х = 945
б) 3 * y — 45 = 44

4. Решите задачу.

Бабушка и внучка слепили 124 пельмени. Сколько пельменей слепили бабушка и сколько внучка, если бабушка лепила в 3 раза быстрее, чем внучка?

Вариант II.

1. Решите примеры.

а) 472 — ( 29 + 124 : 4 ) — 72 : 8 =
б) 182 =
в) 63 =
г) ( 5 + 27 )2 — ( 4 + 12 )2 — 64 =

2. Упростите выражение и найдите его значение при с=12: 19с + 57 — 58с + 29с — 38 + 5с.

3. Решите уравнения:

а) 15 * х = 180
б) 12 * y + 36 = 96

4. Решите задачу.

Инженер и студент отремонтировали 248 приборов. Инженер ремонтировал приборы в 3 раза быстрее, чем студент. Сколько приборов починил каждый?

Вариант III.

1. Решите примеры.

а) 365 + ( 299 — 342 : 2 ) * 5 — 687 : 3 =
б) 172 =
в) 83 =
г) ( 4 + 7 )2 — ( 5 + 23 )2 + 787 =

2. Упростите выражение и найдите его значение при с=12: 47 + 56с — 6с + 34 — 12с.

3. Решите уравнения.

а) 32 * х = 1280
б) 8 * y + 36 = 356

4. Решите задачу.

Портной и его ученик сшили 213 фартуков. Портной работал в 2 раза быстрее, чем его ученик. Сколько фартуков сшил портной, а сколько ученик?

Самостоятельная работа №10 на темы: «Окружность и круг». «Обыкновенные дроби»


Вариант I.

1. Нарисуйте окружность с центром в точке X и радиусом 4 см 6 мм. Нарисуйте отрезок CD так, чтобы он проходил через центр окружности и пересекал ее в точках C и D. Как называются отрезки СX и СD? Определите их длину.

2. Решите задачу.

Оля нашла 26 грибов, из них 18 маслят. Какую часть грибов составляют маслята?

3. Решите задачу.

Рыбаки поймали 112 кг рыбы. Из них 1028 – караси. Сколько карасей поймали рыбаки?

4. Решите задачу.

Коля прочитал 85 страниц журнала, что составило 512 от общего числа страниц. Сколько страниц в журнале?

Вариант II.

1. Нарисуйте окружность с центром в точке Y и радиусом 3 см 8 мм. Нарисуйте отрезок EF так, чтобы он проходил через центр окружности и пересекал ее в точках E и F. Как называются отрезки YE и EF? Определите их длину.

2. Решите задачу.

Коля собрал в корзину 31 фрукт, из них 22 фрукта – это груши. Какую часть собранных фруктов составляют груши?

3. Решите задачу.

Школьники собрали 104 кг овощей. 1326 от общего числа овощей составляют помидоры. Сколько кг помидор собрали школьники?

4. Решите задачу.

Мастер отремонтировал 35 приборов, что составило 512 от общего количества приборов. Сколько всего приборов надо отремонтировать мастеру?

Вариант III.

1. Нарисуйте окружность с центром в точке Z и радиусом 2 см 6 мм. Нарисуйте отрезок GH так, чтобы он проходил через центр окружности и пересекал ее в точках G и H. Как называются отрезки GZ и GH? Определите их длину.

2. Решите задачу.

У Саши есть 29 карандашей. Из них 19 карандашей – это простые карандаши. Какую часть карандашей составляют цветные карандаши?

3. Решите задачу.

Мастер сделал 312 деталей. Из них 324 часть деталей – деревянные. Сколько деревянных деталей сделал мастер?

4. Решите задачу.

Ребята из 5 класса собрали 32 кг ягод. Это составляет 324 от всего количества собранных ягод. Сколько всего ягод было собрано?

Самостоятельная работа №11 на тему: «Сравнение дробей»


Вариант I.

1. Задан луч длиной в 12 единиц. Отметьте на числовом луче:

а) 212 частиб) 612 части23 части54 части

2. Сравните дроби.

а) 2338 и 1618

б) 2145 и 1526

3. Найдите три решения неравенства.

а) 2122< x < 2222

б) 711 < z < 811

4. При каких значениях х:

а) дробь х22 будет правильной?

б) дробь 15х будет неправильной?

Вариант II.

1. Задан луч длиной в 15 единиц. Отметьте на числовом луче:

415 части315 части35 части23 части

2. Сравните дроби.

а) 2634 и 1517

б) 2249 и 1821

3. Найдите три решения неравенства.

а) 1920 < x < 2020

б) 79 < z < 89

4. При каких значениях y:

а) дробь y19 будет правильной?

б) дробь 23y будет неправильной?

Вариант III.

1. Задан луч длиной в 18 единиц. Отметьте на числовом луче:

218 части618 части23 части56 части

2. Сравните дроби.

а) 2631 и 1819

б) 2341 и 1718

3. Найдите три решения неравенства.

а) 910< y < 1010

б) 57 < z < 67

4. При каких значениях z:

а) дробь z29 будет правильной?

б) дробь 13z будет неправильной?

Самостоятельная работа №12 на тему: «Сложение и вычитание обыкновенных дробей»


Вариант I.

1. Решите примеры.

а) 2631 + 1831631;

б) 171255125 + 106125;

в) 1939 + ( 1839639 ) — 1339;

2. Решите уравнения.

а) x + 618 = 1618

б) 1325 — ( y + 625 ) = 425

3. Решите задачу.

Первый спортсмен пробежал 57 км, а второй спортсмен за тоже время пробежал 67 км. На сколько метров больше пробежал первый спортсмен?

4. Решите задачу.

Из мешка взяли 29 части муки, а потом – ещё 39 части. В мешке осталось 14 кг. Сколько кг муки было в мешке?

Вариант II.

1. Решите примеры.

а) 1538 + 12381138;

б) 231928192 + 48192;

в) 1956 + ( 21561256 ) — 1656;

2. Решите уравнения.

а) x — 512 = 312

б) 1823 — ( 723 + y ) = 523

3. Решите задачу.

Расстояние от дачи до пруда равно 35 км, а от дачи до леса равно 45 км. На сколько метров расстояние от дачи до пруда больше, чем расстояние от дачи до леса?

4. Решите задачу.

Из погреба вытащили 312 части картофеля, а потом – ещё 212 части. После этого в погребе осталось 56 кг картофеля. Сколько картофеля было в погребе?

Вариант III.

1. Решите примеры.

а) 1928 + 12281628;

б) 1317611176 + 49176;

в) 2742 + ( 1242642 ) — 1242;

2. Решите уравнения.

а) x + 1223 = 2023

б) 2835 — ( y + 1635 ) = 435

3. Решите задачу.

Расстояние от школы до больницы равно 89 км, а от школы до бассейна равно 49 км. На сколько метров расстояние от школы до больницы больше, чем расстояние от школы до бассейна?

4. Решите задачу.

Из рулона отрезали 38 части ткани, а потом – ещё 28 части. После этого в рулоне осталось 32 метра ткани. Сколько метров ткани было в рулоне?

Самостоятельная работа №13 на тему: «Сложение и вычитание смешанных чисел»


Вариант I.

1. Решите примеры.

а) 4 1928 + 6 1228;

б) 5 13176 — 2 11176;

в) 12 2743 + 3 1243.

2. Решите уравнения.

а) 23 1838 + х =36 1228;

б) 7 1416 — y = 3 1116;

в) y + 18 2753 = 24 1353;

3. Решите задачу.

В первый день в мастерской использовали 23 318 метра проволоки, а во второй день – ещё 18 218 части. После этого в рулоне осталось 32 метра проволоки. Сколько метров проволоки было в рулоне?

Вариант II.

1. Решите примеры.

а) 3 1322 + 3 1222;

б) 8 15126 — 4 15126;

в) 13 2249 + 3 1449.

2. Решите уравнения.

а) 2 1843 + х = 3 443;

б) 17 1519 — y = 12 1219;

в) y — 18 3856 = 24 2756.

3. Решите задачу.

В первый день в школе покрасили 17 523 метра коридора, а во второй день – ещё 23 423 метра. Сколько метров было покрашено за 2 дня?

Вариант III.

1. Решите примеры.

а) 5 1923 + 6 1223;

б) 7 1348 — 3 1148;

в) 82 2578 + 34 1278

2. Решите уравнения.

а) 6 1729 + х = 23 429;

б) 8 15128 — y = 6 12128;

в) y — 18 3847 = 5 2747.

3. Решите задачу.

Фермер убрал 13 613 метра грядки в первый день, а на следующий день – ещё 18 313 метра. После двух дней работы осталось убрать 6 метров. Какова длина грядки?

Самостоятельная работа №14 на темы: «Десятичная запись дробных чисел». «Сравнение десятичных дробей»


Вариант I.

1. Заданные дроби представьте, как десятичные дроби.

а) 5 5910
б) 6 1100

в) 17 1371000

2. Сравните числа.

а) 5,596 и 5,629
б) 7,34 и 7,339
в) 0,684 и 0,6840

3. Переведите из одной единицы измерения в другую.

а) представьте в тоннах: 92 ц; 887 кг; 14 т 12 кг;
б) представьте в квадратных дециметрах: 8 м 2; 57 см 2; 8 м2 77 дм2.

4. Отметьте точки: 0,2; 0,8; 1,1; 2,3; 2,1; 3,7 на числовом отрезке, равном 5 единицам.


Вариант II.

1. Заданные дроби представьте, как десятичные дроби.

а) 18 591000

б) 710

в) 7 137100

2. Сравните числа.

а) 35,97 и 35,971
б) 8,449 и 8,540
в) 0,92 и 0,920

3. Переведите из одной единицы измерения в другую.

а) представьте в тоннах: 3 ц; 239 кг; 23 т 28 кг;
б) представьте в квадратных дециметрах: 13 м 2; 2 см 2; 87 м2 32 дм2.

4. Отметьте точки: 0,5; 0,7; 1,1; 2; 2,3; 3,5 на числовом отрезке, равном 6 единицам.


Вариант III.

1. Заданные дроби представьте, как десятичные дроби.

а) 15 43100

б) 9 231000

в) 510

2. Сравните числа.

а) 29,345 и 29,354
б) 171,89 и 171,889
в) 0,93 и 0,930

3. Переведите из одной единицы измерения в другую.

а) представьте в тоннах: 18 ц; 56 кг; 3 т 9 кг;
б) представьте в квадратных дециметрах: 4 м 2; 23 см 2; 2 м2 56 дм2.

4. Отметьте точки: 0,4; 0,5; 1,4; 1,9; 2,4; 3,0 на числовом отрезке, равном 4 единицам.


Самостоятельная работа №15 на темы: «Сложение и вычитание десятичных дробей». «Округление чисел»


Вариант I.

1. Решите примеры на сложение десятичных дробей.

а) 29,3 + 4,35 =
б) 68,9 + 19,1 =
в) 0,68 + 6,4 =

2. Решите примеры на вычитание десятичных дробей.

а) 35,1 — 13,2 =
б) 37 — 27,3 =
в) 13,28 — 5,327 =

3. Решите задачу:

В первый день плот проплыл 14,8 км, во второй день – на 1 км 700 м больше, чем в первый день. В третий день плот проплыл на 600 м меньше, чем во второй день. Сколько всего км проплыл плот?

4. Округлите:

а) целую часть числа 2539,48190 до сотен, до десятков, до единиц;
б) дробную часть числа 2539,48190 до тысячных, до сотен, до десятков.

Вариант II.

1. Решите примеры на сложение десятичных дробей.

а) 79,3 + 8,15 =
б) 18 + 8,8 =
в) 0,93 + 23,4 =

2. Решите примеры на вычитание десятичных дробей.

а) 48,2 — 4,98 =
б) 96 — 48,6 =
в) 37,67 — 13,168 =

3. Решите задачу.

В первом пакете было 15,7 кг песка, во втором – на 350 г больше, чем в первом. В третьем – на 1200 г меньше, чем в первом. Сколько кг песка в трех пакетах?

4. Округлите:

а) целую часть числа 3462,9470 до сотен, до десятков, до единиц;
б) дробную часть числа 3462,9470 до тысячных, до сотен, до десятков.

Вариант III.

1. Решите примеры на сложение десятичных дробей.

а) 34,3 + 13,11 =
б) 8 + 47,7 =
в) 0,123 + 23,942 =

2. Решите примеры на вычитание десятичных дробей.

а) 69,2 — 7,88 =
б) 91,76 — 18,6 =
в) 8,94 — 5,452 =

3. Решите задачу.

3 дня бабушка пекла блины. В первый день она использовала 1,2 кг муки, во второй день – на 500 г меньше, чем в первый день, а на третий день – на 300 г больше, чем во второй день. Сколько муки она использовала за три дня?

4. Округлите:

а) целую часть числа 4392,73910 до сотен, до десятков, до единиц;
б) дробную часть числа 4392,73910 до тысячных, до сотен, до десятков.

Самостоятельная работа №16 на тему: «Умножение десятичных дробей на натуральные числа»


Вариант I.

1. Выполните умножение.

а) 8,3 * 8 =б) 7,12 * 34 =в) 0,235 * 93 =г) 1,93 * 100 =

2. Найдите значение выражения: х + ( 3,74х — 1,474х ) при х=3; 100; 374; 1000.

3. Решите задачу.

Одновременно навстречу друг другу из двух деревень, расстояние между которыми составляет 45,8 км, вышли пешеходы. Скорость первого пешехода составляет 4,2 км/ч, а скорость второго – 4,5 км/ч. Какое расстояние будет между ними через 4 часа?

4. Решите задачу.

Машина проехала 360 км за 6 часов. Какое расстояние она преодолеет, передвигаясь с той же скоростью, за 14 часа, за 2 13 часа?

Вариант II.

1. Выполните умножение.

а) 7,48 * 12 =б) 3,57 * 7 =в) 0,873 * 87 =г) 1,698 * 1000 =

2. Найдите значение выражения: 5х + ( 6,59х + 2,483х ) при х=5; 100; 324; 1000.

3. Решите задачу.

Одновременно в противоположных направлениях из города выехали 2 машины. Скорость первой машины составляет 54,7 км/ч, а скорость второй – 76,2 км/ч. Какое расстояние будет между ними через 3 часа?

4. Решите задачу.

Велосипедист преодолел 72 км за 3 часа. Какое расстояние он преодолеет, перемещаясь с той же скоростью, за 56 часа, за 2 13 часа?

Вариант III.

1. Выполните умножение.

а) 9,4 * 6 =б) 8,34 * 56 =в) 0,517 * 62 =г) 6,787 * 1000 =

2. Найдите значение выражения: ( 8,45х — 3,594х ) — х при х=8; 100; 843; 1000.

3. Решите задачу.

Одновременно навстречу друг другу из двух городов выехали мотоциклы. Расстояние между городами составляет 234,8 км. Скорость первого мотоциклиста составляет 34,5 км/ч, а скорость второго – 56,2 км/ч. Какое расстояние будет между ними через 2 часа?

4. Решите задачу.

Моторная лодка прошла 24 км за 2 часа. Какое расстояние она пройдет, перемещаясь с той же скоростью, за 14 часа, за 3 13 часа?

Самостоятельная работа №17 на тему: «Деление десятичных дробей на натуральные числа»


Вариант I.

1. Выполните деление.

а) 2,729 : 6 =б) 283,85 : 4 =в) 4 : 13 =г) 0,095 : 10 =

2. Решите уравнения.

а) 5X — 0,4 = 23,6б) 48,2 : Y = 10,4

3. Решите задачу.

За два дня рабочие отремонтировали 3,6 км дороги. В первый день они отремонтировали 1/4 части дороги. Сколько км дороги они отремонтировали во второй день?

4. Решите задачу.

4 класс и 5 класс собирали макулатуру. Пятиклассники собрали в 2 раза больше макулатуры, чем ребята из 4 класса. Вместе они собрали 239,7 кг. Сколько кг собрали ребята из 5 класса и сколько ребята из 4 класса?

Вариант II.

1. Выполните деление.

а) 5,837 : 7 =б) 291,49 : 5 =в) 5 : 18 =г) 0,023 : 10 =

2. Решите уравнения.

а) 8X + 2,8 = 18,6б) 28,1 : Y = 12,4

3. Решите задачу.

За два дня бригада собрала 147,6 кг ягод. В первый день они собрали 4/9 части урожая ягод. Сколько кг ягод они собрали во второй день?

4. Решите задачу.

Две бригады собирали картофель. Первая бригада собрала в 3 раза больше картофеля, чем вторая. Обе бригады вместе собрали 49,6 ц урожая. Сколько центнеров картофеля собрали первая бригада и сколько вторая бригада?

Вариант III.

1. Выполните деление.

а) 4,752 : 9 =б) 472,49 : 6 =в) 7 : 19 =г) 0,044 : 10 =

2. Решите уравнения.

а) 5X + 2,5 = 24б) 14,2 : Y = 3,4

3. Решите задачу.

За 2 дня мотоциклист преодолел 394,1 км. В первый день он проехал 47 части пути. Сколько км он проехал во второй день?

4. Решите задачу.

Мама собрала в 5 раз больше ягод, чем дочка. Вместе они собрали 34,5 кг ягод. Сколько ягод собрала мама и сколько дочка?

Самостоятельная работа №18 на тему: «Среднее арифметическое»


Вариант I.

1. Найдите среднее арифметическое четырех чисел: 4,5; 5,6; 4,9; 5,1.

2. Решите задачу.

В течение часа машина двигалась со скоростью 67,5 км/ч, в течение второго часа – со скоростью 51,6 км/ч. В течение третьего часа её скорость составила 72,3 км/ч. Какова средняя скорость машины? Сколько км она преодолела за 3 часа?

3. Решите задачу.

Среднее арифметическое трех чисел составляет 14,5. Первое число – 14,1, а второе число на 0,8 больше третьего числа. Назовите эти числа.

4. Решите задачу.

Расстояние между двумя деревнями равно 340 км. Автомобиль преодолел половину пути со скоростью 58 км/ч, а вторую половину – со скоростью 49 км/ч. Какова средняя скорость автомобиля на протяжении всего пути?

Вариант II.

1. Найдите среднее арифметическое четырех чисел: 12,3; 12,9; 11,6; 13,1.

2. Решите задачу.

В течение первого часа спортсмен шел со скоростью 11,2 км/ч, в течение второго часа – со скоростью 10,7 км/ч, а в течение третьего часа его скорость составила 9,8 км/ч. Какова средняя скорость спортсмена? Какое расстояние он прошел за 3 часа?

3. Решите задачу.

Среднее арифметическое трех чисел составляет 28,5. Первое число – 28,2, а второе на 0,9 больше третьего числа. Назовите эти числа.

4. Решите задачу.

Расстояние между двумя городами составляет 52 км. Первую половину пути велосипедист передвигался со скоростью 18 км/ч, а вторую половину – со скоростью 22 км/час. Какова средняя скорость велосипедиста на всем протяжении пути?

Вариант III.

1. Найдите среднее арифметическое четырех чисел: 9,1; 9,9; 11,1; 10,7.

2. Решите задачу.

В течение первого часа лодка двигалась со скоростью 15,5 км/ч, во второй час движения её скорость составила 17,4 км/ч, а в течение третьего часа – 12,7 км/ч. Какая средняя скорость лодки? Сколько км она преодолела за 3 часа?

3. Решите задачу.

Среднее арифметическое трех чисел составляет 13,2. Первое число – 13,9, а второе – на 0,7 больше третьего числа. Назовите эти числа.

4. Решите задачу.

Расстояние между двумя деревнями составляет 24 км. Первую половину пути пешеход двигался со скоростью 8 км/ч, а вторую половину – со скоростью 9 км/ч. Какова средняя скорость пешехода на всем протяжении пути?

Самостоятельная работа №19 на тему: «Проценты, задачи на проценты»


Вариант I.

1. Решите задачу.

В спортивной секции занимается 60 учеников, из них 70% составляют девочки. Сколько мальчиков занимается в спортивной секции?

2. Решите задачу.

Ребята четвертых и пятых классов собирали макулатуру. Ребята пятого класса собрали 150 кг макулатуры, что составило 60% общего веса собранной макулатуры. Сколько кг макулатуры собрали ребята?

3. Решите задачу.

Из 15 кг яблок получается 12 кг яблочного пюре. Каков процент выхода пюре из яблок?

Вариант II.

1. Решите задачу.

В 5 классе числится 30 учеников, 60% из них – мальчики. Сколько девочек учится в 5 классе?

2. Решите задачу.

2 бригады собирали помидоры. Первая бригада собрала 320 кг помидор, что составило 40% от общего урожая. Сколько всего помидор собрали обе бригады?

3. Решите задачу.

Из 60 семян взошли 55 растений. Найдите процент всхожести семян.

Вариант III.

1. Решите задачу.

В школе работает 40 человека. Из них 80% – женщины. Сколько мужчин работает в школе?

2. Решите задачу.

Бабушка и внучка собирали яблоки. Бабушка собрала 30 кг яблок, что составило 80% от общего сбора. Сколько кг яблок собрали бабушка и внучка вместе?

3. Решите задачу.

При перемалывании 40 кг зерна получили 25 кг муки. Найдите процент выхода муки.

  Поиск Поиск
  • Школьный помощник
    • математика 5 класс
    • математика 6 класс
    • алгебра 7 класс
    • алгебра 8 класс
    • геометрия 7 класс
    • русский язык 5 класс
    • русский язык 6 класс
    • русский язык 7 класс
  • математика
  • алгебра
  • геометрия
  • русский язык

«»

следующая предыдущая вернуться на предыдущую страницу

Такой страницы нет !!!

  • Популярные запросы
    • Обстоятельство
    • Дополнение
    • Определение
    • Деление дробей
    • Математика 6 класс
    • Русский язык 6 класс
    • Русский язык 7 класс
    • Русский язык 5 класс
    • Алгебра 8 класс
    • Математика 5 класс
    • Алгебра 7 класс
    • Наименьшее общее кратное
    • Буквы о и а в корнях -кос- / -кас-; -гор- / — гар-; -клан- / -клон-; -зар- / -зор-
    • Наибольший общий делитель. Взаимно простые числа
    • Буквы о и а в корнях -кос- / -кас-; -гор- / — гар-; -клан- / -клон-; -зар- / -зор-
    • Деление и дроби
    • Окружность и круг
    • Доли. Обыкновенные дроби
    • Квадратный корень из неотрицательного числа
    • Антонимы. Синонимы
    • Десятичная запись дробных чисел
    • Буквы о – а в корнях -лаг- / -лож-, -рос- / -раст- (-ращ-)

Математика 5 класс — Мир математики

 I.  Чтобы разделить десятичную дробь на натуральное число, нужно делить дробь на это число, как делят натуральные числа и поставить в частном запятую тогда, когда закончится деление целой части.

Примеры. 

Выполнить деление1) 96,25:5;  2) 4,78:4;  3) 183,06:45.

Решение.

Пример 1) 96,25:5.

Делим «уголком» так, как делят натуральные числа. После того, как сносим цифру 2 (число десятых — первая цифра после запятой в записи делимого 96,25), в частном ставим запятую и продолжаем деление.

Ответ19,25.

 

 

Пример 2) 4,78:4.

Делим так, как делят натуральные числа. В частном поставим запятую сразу, как снесем 7 — первую цифру после запятой в делимом 4,78. Продолжаем деление дальше. При вычитании 38-36 получаем 2, но деление не окончено. Как поступаем? Мы знаем, что в конце десятичной дроби можно приписывать нули — от этого значение дроби не изменится. Приписываем нуль и делим 20 на 4. Получаем 5 — деление окончено.

Ответ1,195.

 

Пример 3) 183,06:45.

Делим как 18306 на 45. В частном поставим запятую как только снесем цифру 0 — первую цифру после запятой в делимом 183,06. Так же, как в примере 2) нам пришлось приписать нуль к числу 36 — разности чисел 306 и 270.

Ответ: 4,068.

Вывод: при делении десятичной дроби на натуральное число в частном ставим запятую сразу после того, как сносим цифру в разряде десятых делимого. Обратите внимание: все выделенные красным цветом цифры в этих трех примерах относятся к разряду десятых долей делимого.

Смотрите видео: «Как разделить десятичную дробь на натуральное число».

II. Чтобы разделить десятичную дробь на 10, 100, 1000 и т. д. нужно перенести запятую влево на 1, 2, 3 и т. д. цифр.

Примеры.

Выполнить деление: 1) 41,56:10;  2) 123,45:100;  3) 0,47:100;  4) 8,5:1000;  5) 631,2:10000.

Решение.

Перенос запятой влево зависит от того, сколько в делителе нулей после единицы. Так, при делении десятичной дроби на 10мы будем переносить в делимом запятую влево на одну цифру; при делении на100 — перенесем запятую влево на двецифры; при делении на 1000 перенесем  в данной десятичной дроби запятую на три цифры влево. 

В примерах 3) и 4) пришлось приписать нули перед десятичной дробью, чтобы удобнее было переносить запятую. Однако, приписывать нули можно мысленно, и вы будете это делать, когда хорошо научитесь применять правило II для деления десятичной дроби на 10, 100, 1000 и т. д.

Смотрите видео: «Как разделить десятичную дробь на 10, 100, 1000 и т.д.»

 

I. Чтобы умножить десятичную дробь на натуральное число, нужно умножить ее на это число, не обращая внимания на запятую, и в полученном произведении отделить запятой столько цифр справа, сколько их было после запятой в данной дроби.

Примеры. Выполнить умножение: 1) 1,25·7;   2) 0,345·8;   3) 2,391·14.

Решение.

Смотрите видео: « Как умножить десятичную дробь на натуральное число».

II. Чтобы умножить одну десятичную дробь на другую, нужно выполнить умножение , не обращая внимания на запятые, и в полученном результате отделить запятой справа столько цифр, сколько их было после запятых в обоих множителях вместе.

Примеры. Выполнить умножение: 1) 18, 2·0,09;   2) 3,2·0,065;    3) 0,54·12,3.

Решение. 

Смотрите видео: «Умножение десятичных дробей.»

III. Чтобы умножить десятичную дробь на 10, 100, 1000 и т. д. нужно перенести запятую вправо на 1, 2, 3 и т. д. цифр.

Примеры. Выполнить умножение: 1) 3,25·10; 2) 0,637·100; 3) 4,307·1000; 4) 2,04·1000; 5) 0,00031·10000.

Решение.

Смотрите видео: «Умножение десятичных дробей на 10, 100, 1000 и т. д.»

IV. Чтобы умножить десятичную дробь на 0,1; 0,01; 0,001 и т. д. нужно перенести запятую влево на 1, 2, 3 и т. д. цифр. 

Примеры. Выполнить умножение: 1) 28,3·0,1; 2) 324,7·0,01; 3) 6,85·0,01; 4) 6179,5·0,001;  5) 92,1·0,0001.

Решение.

Смотрите видео: «Умножение десятичных дробей на 0,1; 0,001; 0,0001 и т. д.»

Умножение и деление натуральных чисел — правила и примеры для 5 класса

Одними из самых простых операций в математике являются умножение и деление натуральных чисел. В 5 классе после изучения арифметических действий школьников учат приёмам нахождения произведения и частного. Это знания, на которых базируется не только алгебра, геометрия, физика, химия, информатика, но даже и гуманитарные науки. Пожалуй, эти умения используются на практике больше любых других, полученных при обучении в средней школе.

Общие сведения

Математические вычисления сопровождают человека на всём протяжении его жизни. Когда произносится слово «число», имеется в виду определённый символ, определяющий количество чего-либо. Существуют различного вида выражения, например, целые, дробные, логарифмические. Но самыми простыми являются натуральные. Своё название они получили из-за применения в повседневной жизни. Их используют для счёта и определения порядка.

Таким образом, под натуральными числами понимают выражения, применяемые для определения количества любого физического объекта или присваивания порядкового номера. Например, 3, 1789, 9876, 100009. Если такие числа расположить в порядке увеличения, этот ряд называют натуральным. Последовательность 2, 3, 4, 5 будет именно такой. Нужно отметить, что натуральный ряд бесконечен, наибольшего значения в нём не существует.

Есть несколько систем счисления. В зависимости от неё, для обозначения используется различный набор символов. В России, США, европейских странах применяют арабскую систему. При этом в повседневности используется десятичная разрядность, то есть для записи чисел берут знаки от 0 до 9.

С числами можно выполнять любые действия. Их складывают, вычитают, перемножают и делят. Кроме этого, возводят в степень, извлекают из-под корня, логарифмируют и дифференцируют.

К основным свойствам натуральных чисел относят:

  • коммутативность при прибавлении;
  • бинарность операции умножения;
  • ассоциативность при сложении и умножении;
  • дистрибутивность произведения относительно сложения.

Эти свойства важны. На них часто опираются при решении примеров на умножение и деление в 5 классе средней школы. Каждая запись числа состоит из определённого количества разрядов. По сути, она составляет совокупность разрядных слагаемых. В качестве единиц принимают десятки. Любое натуральное выражение можно представить в виде суммы таких чисел. Например, 89 состоит из 8 десятков и 9 единиц. Значит, равенство 89 = 80 + 9 будет справедливым.

Неизвестную натуральную цифру принято обозначать маленькой латинской буквой эн (n). Интересно то, что пересчитать все числа невозможно.

Их количество бесконечно. Самое большое, которое удалось определить называется гугол. Оно содержит 100 нулей и является мерой атомов в физике.

Принцип умножения

Операция умножения подразумевает действие, заменяющее собой многократное сложение. Один из аргументов называют множимым, а другой множителем. Результатом умножения является произведение. Найти его довольно просто, если знать свойства операции.

К достаточным правилам, зная которые можно найти произведение любых чисел, относят:

  • Сочетательное — если при умножении произведения на любое число изменить порядок аргументов, результат не изменится. В буквенном виде закон имеет вид: a * b * c = a * c * b. Это правило можно доказать на опыте. Если взять квадраты размером 1 на 1 и построить из них блок 6 на 6, то фактически это будет перемножение 1 * 6 = 6. Полученный прямоугольник можно объединить с аналогичными 3. То есть 3 * 1 = 3. Общее число квадратов получится 1 * 6 * 3 = 18. Если же последовательность сборки изменить, сначала собрать предмет из трёх блоков, а потом к ним добавить 6, результат не изменится.
  • Распределительное — при выполнении действия над суммой и числом, можно отдельно каждый член выражения помножить на множитель, а затем результаты сложить. В математической записи правило выглядит так: a * (b + c) = a * b + a * c. По-другому операция называется раскрытием скобок. Это правило аналогично и для вычитания. Но при этом есть нюанс, что умножение выполняют сначала на уменьшаемое, а затем вычитаемое, и из первого вычитают второе.
  • Умножения на 0. Любое натуральное число при умножении на 0 даст в ответе 0. Справедливо и обратное утверждение.
  • Для умножения до 100 существует специальная таблица, которую необходимо знать наизусть.

    Следует также понимать, что при увеличении числа в десятки раз ответ увеличится на число нулей, стоящих в умножаемой цифре. Например, 34 * 10 = 340; 980 * 1000 = 980000. Так, выполняется сколь угодно сложное перемножение и для чисел большего десятка.

    Произведение часто находят методом «столбик». Суть способа состоит в том, что аргументы записывают один под одним. При этом самая правая цифра верхнего числа должна стоять над самой правой нижнего. Далее выполняют поразрядное умножение начиная с младших членов. Если при этом образуется высший разряд, он прибавляется к перемножаемому.

    Результат умножения следующего десятка сдвигается на единицу влево. Далее, складывают полученные результаты и получают искомое произведение.

    Выполнение деления

    Между нахождением частного и произведения существует тесная взаимосвязь. Особенно она просматривается при решении примеров на деление в 5 классе. По сути, эти 2 действия являются обратными друг другу. Математическим языком это можно описать как b * a = c → b = c / a. Эта зависимость в дальнейшем довольно сильно помогает решать сложные многозначные уравнения.

    Существует несколько способов поиска частного:

  • Последовательное вычитание. Нужно число разделить на другое. Чтобы найти ответ, понадобится из делимого вычитать делитель до тех пор, пока в ответе не получится 0. Затем следует подсчитать количество вычитаний. Это число и будет искомым ответом. На самом деле этот способ используется редко из-за своей громоздкости.
  • Представление в виде произведения. При решении примеров иногда удобно делимое разложить на множители, причём так, чтобы один из них легко можно было разделить на делитель. Например, 560 / 56 = (56 * 10) / 56 = 10.
  • Использование метода «уголок». Это наиболее часто применяемый способ. Делимое с делителем записывают в строчку, разделяя горизонтальной чертой. Вначале, сравнивая цифры, определяют неполное частное. Если в числе, что стоит справа, количество единиц меньше, добавляют следующий разряд. Затем подбирают такой множитель, чтобы при его умножении на делитель ответ не превышал выбранную часть делимого. Полученный результат записывают под низом делителя. Это будет первая цифра частного. Далее, от делимого вычитают результат умножения. Такие действия повторяют до тех пор, пока не получится 0.
  • Существуют методы, позволяющие проверить, насколько правильно найдено частное. Для этого нужно полученный ответ перемножить с делителем. Например, 12 / 4 = 3. Отсюда 3 * 4 = 12. Все три члена идентичные, значит, ответ найден верно.

    Следует знать, что есть приёмы, позволяющие облегчить выполнение действия. При нахождении результата деления, когда нужно найти частное двух одинаковых чисел, в ответе будет единица: 345/ 345 = 78 / 78 = 89976 / 89976 = 1.

    При этом 0, разделённый на любое число, даст в ответе 0. Делить же на него нельзя: выражение не будет иметь смысла.

    Решение примеров

    В 5 классе на математике всегда ученикам преподаватель предлагает решить определённые задания. Это нужно, чтобы школьник закрепил полученные теоретические знания и научился их применять на практике. Существуют сборники примеров по математике за 5 класс на умножение и деление для самостоятельной проработки. Прорешав успешно оттуда задачи, любой учащийся сможет утверждать, что он разобрался в теме.

    Вот некоторые из примеров, содержащиеся в таких задачниках:

  • Найти произведение выражения: 5 * 2 * (3 + 6) — 17. Вначале нужно выполнить операцию умножения, затем раскрыть скобки и от полученного результата отнять 17. Произведение пятёрки на двойку — это стандартное действие. Ответ операции нужно знать наизусть или сложить 2 раза цифру 5. Раскрыть скобки поможет распределительный закон. В итоге решение будет иметь следующий вид: 5 * 2 * (3 + 6) — 17 = 10 * (3 + 6) — 17 = 10 * 3 + 10 * 6 — 17 = 30 + 60 — 17 = 90 — 17 = 73.
  • Вычислить ответ: 450 :10 — 12 * 3 + 45: 45. Согласно правилам, сначала выполняют деление, а уже после вычитание и сложение. Определяя частное для первого члена, можно увидеть, что 450 = 45 * 10. В последнем же выражении число делится само на себя, значит, частное будет равно 1. Чтобы 12 умножить на 3, нужно сначала тройку перемножить с двойкой, а потом с единицей. Если это сделать, в ответе получится 36. Таким образом, решить пример можно так: 450: 10 — 12 * 3 + 45: 45 = 45 * 10: 10 — 12 * 3 + 1 = 45 — 12 * 3 + 1 = 45 — 36 + 1 = 45 — 37 = 8.
  • Решить уравнение 4 * n = 144. Исходя из смысла деления, можно записать n = 144: 4. Действие в столбик будет выглядеть так: 4 * 3 = 12, 3 пишется в частное, 14 — 12 = 2, сносится четвёрка и получается 24. Подбирается вторая цифра 4 * 6 = 24. Значит, в ответе получится n = 26.
  • С автобазы выехали 8 машин. В каждой из них было по 3 тонны груза. Каждая тонна размещалась в 42 ящиках. Сколько всего тары было отправлено со склада? Решение будет состоять из двух этапов. На первом нужно подсчитать, сколько ящиков было в каждом грузовике: 42 * 3 = 126. На втором определить число тары: 126 * 8 = 1008. Ответ нужно будет написать так: всего со склада было отправлено 1008 ящиков.
  • В начальных классах учителя при решении задач не разрешают пользоваться калькуляторами. Это необходимая мера.

    Ведь чтобы научиться, важно не только понимать суть действий, но и набраться необходимого опыта. При этом обязательно нужно наизусть выучить таблицу умножения.

    Предыдущая

    МатематикаНеправильные дроби — примеры для 5 класса с решением и объяснением

    Следующая

    МатематикаКонъюнкция и дизъюнкция — правила и примеры решения в математике

    Задачи и примеры по математике за 5 класс: тренажер по математике для 5 класса онлайн

    Ваш ребенок с трудом решает задачи и не может освоить десятичные дроби? Мы поможем решить эту проблему. Предлагаем пройти тест по математике за 5 класс на интеллектуальной платформе Skills4u абсолютно бесплатно. Всего за несколько минут вы сможете оценить уровень владения материалом и получите рейтинг, составленный на основе правильных ответов.

    Наша платформа предлагает интерактивные тесты по математике (5 класс), основанные на интеллектуальном алгоритме. Суть в том, что выдача заданий происходит на основе анализа результатов опроса. Это персонализированный подход, позволяющий учесть уровень подготовки каждого ученика и предложить примеры, которые ему по силам. При этом каждый раз задания могут быть иными, постепенно усложняясь по мере усвоения материала.

    Посещает ли ваш ребенок частную или государственную школу, занимается дополнительно онлайн или ходит к репетитору – тестирование по математике за 5 класс выявит пробелы в знаниях и поможет сформировать прочные навыки решения любых задач. Весь материал разбит по темам. Вы можете выбрать ту, в которой ваш ребенок испытывает затруднения, или сделать полный срез успеваемости, чтобы составить полную картину. Пробный онлайн тест по математике за 5 класс вы можете пройти совершенно бесплатно прямо сейчас. Платформа оценит ответы, предложит исправить ошибки и составит рейтинг ученика.

    Для формирования устойчивого навыка, доходящего до автоматизма, необходимо решать примеры по вычислительным навыкам за 5 класс по математике в течение нескольких последующих дней для закрепления материала. Для этого следует оформить доступ к образовательной платформе Skills4u, выбрав один из планов: на 1 месяц, на полгода или на 12 месяцев. Стоимость невелика, а результат будет очень скоро заметен.

    Если ваш ребенок будет регулярно проходить онлайн тестирование за 5 класс по математике на интерактивной платформе, он сможет быстро и безошибочно решать самые сложные примеры и уравнения, производить вычисления с десятичными дробями. Разумеется, гарантированный положительный результат дает только продолжительная подписка, но даже месяц занятий позволит сдвинуться с мертвой точки.

    Увлекательная форма без дополнительных письменных заданий, интеллектуальный алгоритм выдачи примеров, позволяющий учитывать уровень подготовки каждого ребенка – вот залог успеха нашей программы. Присоединяйтесь!

    Материалы для организации дистанционного обучения. Математика (5-6 классы)

    Класс Название урока Ссылка на учебные материалы
    5 Ряд натуральных чисел. Десятичная система записи натуральных чисел https://resh.edu.ru/subject/lesson/7719/main/234018/
    5 Сравнение натуральных чисел https://resh.edu.ru/subject/lesson/7718/main/235258/
    5 Сравнение именованных чисел https://interneturok.ru/lesson/matematika/5-klass/bnaturalnye-chislab/izmerenie-velichin-edinitsy-izmereniya?block=player
    5 Измерение отрезков https://resh.edu.ru/subject/lesson/7740/main/234855/
    5 Длина отрезка https://resh.edu.ru/subject/lesson/6914/main/235937/
    5 Прямая, луч, отрезок https://resh.edu.ru/subject/lesson/7741/main/235227/
    5 Координатный луч https://resh.edu.ru/subject/lesson/7756/main/234421/
    5 Представление натуральных чисел на координатном луче https://resh.edu.ru/subject/lesson/7738/main/233801/
    5 Сложение натуральных чисел. Законы сложения https://resh.edu.ru/subject/lesson/7723/main/
    5 Арифметические действия с натуральными числами https://mosobr.tv/release/7856
    5 Вычитание https://resh.edu.ru/subject/lesson/7717/main/235289/
    5 Правила вычитания суммы из числа и числа из суммы https://znaika.ru/catalog/5-klass/matematika/Svoystva-vychitaniya-naturalnykh-chisel.html
    5 Решение текстовых задач с помощью сложения и вычитания https://resh.edu.ru/subject/lesson/7716/main/233832/
    5 Числовые выражения https://resh.edu.ru/subject/lesson/7708/main/266123/
    5 Углы. Измерение углов https://resh.edu.ru/subject/lesson/589/
    5 Углы. Измерение углов https://resh.edu.ru/subject/lesson/7735/main/234886/
    5 Многоугольники https://resh.edu.ru/subject/lesson/7727/main/267638/
    5 Треугольники https://resh.edu.ru/subject/lesson/7734/main/234917/
    5 Четырёхугольники https://resh.edu.ru/subject/lesson/7733/main/233522/
    5 Симметрия относительно точки, относительно прямой и относительно плоскости https://resh.edu.ru/subject/lesson/983/
    5 Умножение. Законы умножения https://resh.edu.ru/subject/lesson/7722/main/235069/
    5 Распределительный закон https://resh.edu.ru/subject/lesson/7724/main/266247/
    5 Умножение. Законы умножения https://resh.edu.ru/subject/lesson/7712/main/235041/
    5 Деление нацело https://resh.edu.ru/subject/lesson/7712/main/235041/
    5 Деление с остатком https://resh.edu.ru/subject/lesson/7709/main/235165/
    5 Степень с натуральным показателем https://resh.edu.ru/subject/lesson/7713/main/234793/
    5 Решение текстовых задач с помощью умножения и деления https://resh.edu.ru/subject/lesson/7711/main/234760/
    5 Площадь прямоугольника. Единицы площади https://resh.edu.ru/subject/lesson/7732/main/233181/
    5 Прямоугольный параллелепипед https://resh.edu.ru/subject/lesson/7731/main/234669/
    5 Объём прямоугольного параллелепипеда. Единицы объёма https://resh.edu.ru/subject/lesson/7730/main/272356/
    5 Объём прямоугольного параллелепипеда https://resh.edu.ru/subject/lesson/7753/main/234824/
    5 Понятие дроби https://resh.edu.ru/subject/lesson/7782/main/233643/
    5 Сравнение обыкновенных дробей https://resh.edu.ru/subject/lesson/16/
    5 Задачи на дроби (нахождение части целого) https://resh.edu.ru/subject/lesson/7780/main/234203/
    5 Дроби и деление натуральных чисел https://znaika.ru/catalog/5-klass/matematika/Delenie-i-drobi.html
    5 Нахождение целого по его части https://resh.edu.ru/subject/lesson/7779/main/233956/
    5 Задачи на дроби (нахождение части целого) https://resh.edu.ru/subject/lesson/7780/main/234203/
    5 Сложение дробей с одинаковым знаменателем https://resh.edu.ru/subject/lesson/7774/main/233212/
    5 Понятие смешанной дроби https://resh.edu.ru/subject/lesson/7761/main/233026/
    5 Сложение смешанных дробей https://resh.edu.ru/subject/lesson/7760/main/233336/
    5 Вычитание смешанных дробей https://resh.edu.ru/subject/lesson/7759/main/233615/
    5 Понятие положительной десятичной дроби https://resh.edu.ru/subject/lesson/6903/main/235413/
    5 Сравнение положительных десятичных дробей https://resh.edu.ru/subject/lesson/6902/main/236096/
    5 Округление натуральных чисел. Теоретическая часть https://resh.edu.ru/subject/lesson/23/
    5 Округление десятичных дробей https://resh.edu.ru/subject/lesson/27/
    5 Чтение и запись десятичных дробей https://video-ypoku.com/dlya-detej-i-roditelej/shkolnikam/1383-matematika-5-klass-desyatichnye-drobi
    5 Сложение положительных десятичных дробей https://resh.edu.ru/subject/lesson/6901/main/236064/
    5 Вычитание положительных десятичных дробей https://resh.edu.ru/subject/lesson/6900/main/236033/
    5 Сложение и вычитание десятичных дробей https://resh.edu.ru/subject/lesson/719/
    5 Умножение и деление десятичной дроби на 10, 100, 1000 https://resh.edu.ru/subject/lesson/720/
    5 Умножение положительных десятичных дробей. Часть 1 https://resh.edu.ru/subject/lesson/6898/main/235559/
    5 Умножение положительных десятичных дробей. Часть 2 https://resh.edu.ru/subject/lesson/6897/main/236204/
    5 Деление натуральных чисел https://mosobr.tv/release/7932
    5 Деление десятичной дроби на целое число https://resh.edu.ru/subject/lesson/722/
    5 Умножение и деление десятичной дроби на 10, 100, 1000 https://resh.edu.ru/subject/lesson/720/
    5 Деление положительных десятичных дробей. Часть 1 https://resh.edu.ru/subject/lesson/6896/main/236240/
    5 Деление положительных десятичных дробей. Часть 2 https://resh.edu.ru/subject/lesson/6895/main/237500/
    5 Среднее арифметическое https://resh.edu.ru/subject/lesson/715/
    5 Среднее арифметическое нескольких чисел https://resh.edu.ru/subject/lesson/22/
    5 Понятие о проценте https://resh.edu.ru/subject/lesson/6846/main/237181/
    5 Проценты. Вычисление процентов от числа и числа по известному проценту, выражение отношения в процентах https://resh.edu.ru/subject/lesson/1060/
    5 Задачи на проценты. Часть 1 https://resh.edu.ru/subject/lesson/6848/main/237738/
    5 Задачи на проценты. Часть 2 https://resh.edu.ru/subject/lesson/6839/main/237149/
    5 Занимательные задачи по теме «Смешанные дроби» https://resh.edu.ru/subject/lesson/7752/main/233553/
    5 Итоговое обобщение и систематизация знаний по темам «Делимость натуральных чисел» https://resh.edu.ru/subject/lesson/7790/main/234080/
    5 Итоговое обобщение и систематизация знаний по темам «Обыкновенные дроби и смешанные дроби» https://resh.edu.ru/subject/lesson/7789/main/266061/
    5 Единицы длины https://uchebnik.mos.ru/catalogue/material_view/atomic_objects/687915
    5 Шкала. Примеры шкал https://infourok.ru/videouroki/2947
    5 Буквенное выражение и его значение https://infourok.ru/videouroki/2952
    5 Формулы https://infourok.ru/videouroki/2922
    5 Уравнение https://uchebnik.mos.ru/catalogue/material_view/atomic_objects/4369029
    5 Биссектриса угла https://uchebnik.mos.ru/catalogue/material_view/atomic_objects/2680471
    5 Равные фигуры https://uchebnik.mos.ru/catalogue/material_view/atomic_objects/4909803
    5 Прямоугольник. Периметр прямоугольника https://uchebnik.mos.ru/catalogue/material_view/atomic_objects/1299188
    5 Порядок действий https://infourok.ru/videouroki/2920
    5 Понятие площади https://uchebnik.mos.ru/catalogue/material_view/atomic_objects/856976
    5 Пирамида https://uchebnik.mos.ru/catalogue/material_view/atomic_objects/3149941
    5 Ломаная. Длина ломаной https://uchebnik.mos.ru/moderator_materials/material_view/atomic_objects/4454434
    5 Развёртки геометрических фигур https://uchebnik.mos.ru/moderator_materials/material_view/atomic_objects/5795070
    5 Округление чисел. Прикидки https://uchebnik.mos.ru/catalogue/material_view/atomic_objects/5206540
    5 Умножение десятичных дробей на 0,1, 0,01, 0,001 и т. д. https://uchebnik.mos.ru/catalogue/material_view/atomic_objects/439985
    5 Деление десятичной дроби на 0,1, 0,01, 0,001 и т. д. https://znaika.ru/catalog/5-klass/matematika/Delenie-na-desyatichnuyu-drob.html
    6 Делители натурального числа https://resh.edu.ru/subject/lesson/694/
    6 Признаки делимости https://resh.edu.ru/subject/lesson/7750/main/234576/
    6 Простые и составные числа https://resh.edu.ru/subject/lesson/7749/main/234979/
    6 Наибольший общий делитель (НОД) https://resh.edu.ru/subject/lesson/7747/main/233739/
    6 Наименьшее общее кратное (НОК) https://resh.edu.ru/subject/lesson/7746/main/234266/
    6 Равенство дробей https://resh.edu.ru/subject/lesson/7781/main/269492/
    6 Понятие дроби https://resh.edu.ru/subject/lesson/7782/main/233643/
    6 Приведение дробей к общему знаменателю. Сокращение дробей https://resh.edu.ru/subject/lesson/7777/main/233115/
    6 Приведение дробей к общему знаменателю. Сокращение дробей https://resh.edu.ru/subject/lesson/7778/main/233708/
    6 Сравнение дробей https://resh.edu.ru/subject/lesson/7776/main/233243/
    6 Сравнение дробей. Сравнение с единицей. Сравнение остатков до единицы https://resh.edu.ru/subject/lesson/7775/main/233398/
    6 Сложение дробей с одинаковым знаменателем https://resh.edu.ru/subject/lesson/7774/main/233212/
    6 Сложение дробей с разными знаменателями https://resh.edu.ru/subject/lesson/7773/main/233057/
    6 Вычитание дробей https://resh.edu.ru/subject/lesson/7771/main/233150/
    6 Решение задач с использованием вычитания дробей https://resh.edu.ru/subject/lesson/7770/main/233925/
    6 Умножение натурального числа на дробь https://resh.edu.ru/subject/lesson/7769/main/234483/
    6 Законы умножения. Распределительный закон https://resh.edu.ru/subject/lesson/7767/main/234545/
    6 Нахождение части целого и целого по его части https://resh.edu.ru/subject/lesson/7764/main/233584/
    6 Решение задач на применение умножения дроби на натуральное число и умножение дробей https://resh.edu.ru/subject/lesson/7768/main/234142/
    6 Умножение и деление обыкновенной дроби на обыкновенную дробь https://resh.edu.ru/subject/lesson/710/
    6 Деление дробей https://resh.edu.ru/subject/lesson/7766/main/234948/
    6 Свойства деления https://znaika.ru/catalog/5-klass/matematika/Delenie-naturalnykh-chisel-i-ego-svoystva.html
    6 Понятие положительной десятичной дроби https://resh.edu.ru/subject/lesson/6903/main/235413/
    6 Бесконечные периодические десятичные дроби https://resh.edu.ru/subject/lesson/6915/main/236435/
    6 Разложение положительной обыкновенной дроби в конечную десятичную дробь https://resh.edu.ru/subject/lesson/6919/main/237273/
    6 Отношение чисел и величин https://resh.edu.ru/subject/lesson/6844/main/235847/
    6 Пропорции https://resh.edu.ru/subject/lesson/6841/main/237211/
    6 Проценты. Вычисление процентов от числа и числа по известному проценту, выражение отношения в процентах https://resh.edu.ru/subject/lesson/1060/
    6 Прямая и обратная пропорциональность https://resh.edu.ru/subject/lesson/6849/main/237769/
    6 Деление числа в данном отношении https://resh.edu.ru/subject/lesson/6842/main/235816/
    6 Окружность и круг. Сфера и шар https://resh.edu.ru/subject/lesson/588/
    6 Длина окружности. Площадь круга https://resh.edu.ru/subject/lesson/6913/main/236498/
    6 Куб, шар, пирамида, цилиндр, конус https://resh.edu.ru/subject/lesson/557/
    6 Столбчатые и круговые диаграммы https://resh.edu.ru/subject/lesson/1228/
    6 Отрицательные целые числа https://resh.edu.ru/subject/lesson/6872/main/237087/
    6 Координатная ось. Часть 1 https://resh.edu.ru/subject/lesson/6912/main/236158/
    6 Представление целых чисел на координатной оси https://resh.edu.ru/subject/lesson/6866/main/236343/
    6 Координатная ось. Часть 2 https://resh.edu.ru/subject/lesson/6920/main/236020/
    6 Противоположные числа. Модуль числа https://resh.edu.ru/subject/lesson/6862/main/237056/
    6 Сравнение целых чисел https://resh.edu.ru/subject/lesson/6861/main/237025/
    6 Сложение целых чисел. Часть 1 https://resh.edu.ru/subject/lesson/6863/main/236994/
    6 Сложение целых чисел. Часть 2 https://resh.edu.ru/subject/lesson/6864/main/236963/
    6 Сложение целых чисел. Часть 3 https://resh.edu.ru/subject/lesson/6865/main/236933/
    6 Законы сложения целых чисел https://resh.edu.ru/subject/lesson/6860/main/237335/
    6 Рациональное вычисление значений арифметических выражений с применением вычитания, как алгебраического сложения. Часть 1 (теория) https://resh.edu.ru/subject/lesson/1380/
    6 Разность целых чисел. Часть 1 https://resh.edu.ru/subject/lesson/6859/main/237552/
    6 Разность целых чисел. Часть 2 https://resh.edu.ru/subject/lesson/6858/main/236901/
    6 Произведение целых чисел. Часть 1 https://resh.edu.ru/subject/lesson/6857/main/236870/
    6 Произведение целых чисел. Часть 2 https://resh.edu.ru/subject/lesson/6856/main/237301/
    6 Произведение целых чисел. Часть 1 https://resh.edu.ru/subject/lesson/6857/main/236870/
    6 Распределительный закон https://resh.edu.ru/subject/lesson/6871/main/236746/
    6 Раскрытие скобок и заключение в скобки https://resh.edu.ru/subject/lesson/6855/main/235382/
    6 Частное целых чисел. Часть 1 https://resh.edu.ru/subject/lesson/6869/main/237521/
    6 Частное целых чисел. Часть 2 https://resh.edu.ru/subject/lesson/6870/main/236808/
    6 План и этапы решения задачи. Анализ решения, Проверка решения, проверка обратным действием https://resh.edu.ru/subject/lesson/608/
    6 Зависимости между величинами: скорость, время, расстояние; производительность, время, работа; цена, количество, стоимость https://resh.edu.ru/subject/lesson/346/
    6 Использование таблиц, схем, чертежей, других средств представления данных при решении задачи https://resh.edu.ru/subject/lesson/340/
    6 Построение фигур, симметричных относительно заданной точки или прямой. Часть 1 https://resh.edu.ru/subject/lesson/1392/
    6 Построение фигур, симметричных относительно заданной точки или прямой. Часть 2. https://resh.edu.ru/subject/lesson/1120/
    6 Декартова система координат на плоскости https://resh.edu.ru/subject/lesson/6921/main/236560/
    6 Столбчатые диаграммы. Графики https://resh.edu.ru/subject/lesson/6911/main/235706/
    6 Построение столбчатых диаграмм. Чтение графиков https://resh.edu.ru/subject/lesson/6922/main/236622/
    6 Обобщение и систематизация знаний по теме «Декартова система координат на плоскости» https://resh.edu.ru/subject/lesson/6917/main/236653/
    6 Занимательные задачи на проценты https://resh.edu.ru/subject/lesson/6918/main/235909/
    6 Сложные задачи на проценты https://resh.edu.ru/subject/lesson/6910/main/236839/
    6 Обобщение и систематизация знаний по темам «Десятичные дроби, проценты, решение задач на проценты» https://resh.edu.ru/subject/lesson/6923/main/236777/
    6 Случайные события https://znaika.ru/catalog/6-klass/matematika/Pervoe-znakomstvo-s-ponyatiem-veroyatnost.html
    6 Вероятность случайного события https://znaika.ru/catalog/6-klass/matematika/Pervoe-znakomstvo-s-ponyatiem-veroyatnost.html
    6 Арифметические действия с рациональными числами https://clck.ru/MWVCo
    6 Свойства умножения рациональных чисел https://uchebnik.mos.ru/catalogue/material_view/atomic_objects/4033089
    6 Свойства вычитания рациональных чисел https://uchebnik.mos.ru/catalogue/material_view/atomic_objects/4368360
    6 Коэффициент https://infourok.ru/videouroki/2988
    6 Перпендикулярные прямые https://infourok.ru/videouroki/2991
    6 Параллельные прямые https://infourok.ru/videouroki/2992

    5 класс. Математика. Десятичные дроби — Умножение и деление десятичных дробей

    Комментарии преподавателя

    Упраж­не­ние. Как умно­жить число 25,78 на 10?

    Де­ся­тич­ная за­пись дан­но­го числа – это со­кра­щен­ная за­пись суммы. Необ­хо­ди­мо рас­пи­сать ее более по­дроб­но:

    Таким об­ра­зом, нужно умно­жить сумму. Для этого можно про­сто умно­жить каж­дое сла­га­е­мое:

    Вы­хо­дит, что.

    Можно сде­лать вывод, что умно­жить де­ся­тич­ную дробь на 10 очень про­сто: нужно за­пя­тую сдви­нуть впра­во на одну по­зи­цию.

    Упраж­не­ние. Умно­жить 25,486 на 100.

    Умно­жить на 100 – это то же самое, что и умно­жить два раза на 10. Иными сло­ва­ми, необ­хо­ди­мо сдви­нуть за­пя­тую впра­во два раза:

    Деление на 10, 100…

    Упраж­не­ние. Раз­де­лить 25,78 на 10.

    Как и в преды­ду­щем слу­чае, необ­хо­ди­мо пред­ста­вить число 25,78 в виде суммы:

    Так как нужно по­де­лить сумму, то это эк­ви­ва­лент­но де­ле­нию каж­до­го сла­га­е­мо­го:

    Итак, .

    Вы­хо­дит, чтобы раз­де­лить на 10, нужно за­пя­тую сдви­нуть влево на одну по­зи­цию. На­при­мер:

    Упраж­не­ние. Раз­де­лить 124,478 на 100.

    Раз­де­лить на 100 – это то же самое, что два раза раз­де­лить на 10, по­это­му за­пя­тая сдви­га­ет­ся влево на 2 по­зи­ции:

    Правило умножения и деления на 10, 100,..

    Если де­ся­тич­ную дробь нужно умно­жить на 10, 100, 1000 и так далее, нужно за­пя­тую сдви­нуть впра­во на столь­ко по­зи­ций, сколь­ко нулей у мно­жи­те­ля.

    И на­о­бо­рот, если де­ся­тич­ную дробь нужно по­де­лить на 10, 100, 1000 и так далее, нужно за­пя­тую сдви­нуть влево на столь­ко по­зи­ций, сколь­ко нулей у мно­жи­те­ля.

    Примеры, когда необходимо перенести запятую, а цифр уже не осталось.

    Умно­жить на 100 зна­чит сдви­нуть за­пя­тую впра­во на две по­зи­ции.

    После сдви­га можно об­на­ру­жить, что после за­пя­той уже нет цифр, а это зна­чит, что дроб­ная часть от­сут­ству­ет. Тогда и за­пя­тая не нужна, число по­лу­чи­лось целое.

    При­мер 2

    Сдви­гать нужно на 4 по­зи­ции впра­во. Но цифр после за­пя­той всего две. Стоит вспом­нить, что для дроби 56,14 есть эк­ви­ва­лент­ная за­пись.

    Те­перь умно­жить на 10 000 не со­став­ля­ет труда:

    Если не очень по­нят­но, по­че­му можно до­пи­сать два нуля к дроби в преды­ду­щем при­ме­ре, то до­пол­ни­тель­ное видео по ссыл­ке смо­жет по­мочь в этом.

    Эк­ви­ва­лент­ные де­ся­тич­ные за­пи­си

    За­пись 52 озна­ча­ет сле­ду­ю­щее:

    Если впе­ре­ди по­ста­вить 0, по­лу­чим за­пись 052. Эти за­пи­си эк­ви­ва­лент­ны.

    Можно ли по­ста­вить два нуля впе­ре­ди? Да, эти за­пи­си эк­ви­ва­лент­ны.

    Те­перь по­смот­рим на де­ся­тич­ную дробь:

    Если при­пи­сать ноль, то по­лу­ча­ет­ся:

    Эти за­пи­си эк­ви­ва­лент­ны. Ана­ло­гич­но можно при­пи­сать несколь­ко нулей.

    Таким об­ра­зом, к лю­бо­му числу можно при­пи­сать несколь­ко нулей после дроб­ной части и несколь­ко нулей перед целой ча­стью. Это будут эк­ви­ва­лент­ные за­пи­си од­но­го и того же числа.

    При­мер 3

    Так как про­ис­хо­дит де­ле­ние на 100, то необ­хо­ди­мо сдви­нуть за­пя­тую на 2 по­зи­ции влево. Слева от за­пя­той не оста­лось цифр. Целая часть от­сут­ству­ет. Такую за­пись часто ис­поль­зу­ют про­грам­ми­сты. В ма­те­ма­ти­ке же, если целой части нет, то ста­вят ноль вме­сто нее.

    При­мер 4

    Сдви­гать нужно влево на три по­зи­ции, но по­зи­ций всего две. Если перед чис­лом на­пи­сать несколь­ко нулей, то это будет эк­ви­ва­лент­ная за­пись.

    То есть при сдви­ге влево, если цифры кон­чи­лись, необ­хо­ди­мо вос­пол­нить их ну­ля­ми.

    При­мер 5

    В дан­ном слу­чае стоит пом­нить, что за­пя­тая все­гда стоит после целой части. Тогда:

    Умножение и деление на 0,1,  0,01…

    Умно­же­ние и де­ле­ние на числа 10, 100, 1000 – очень про­стая про­це­ду­ра. Точно так же дело об­сто­ит и с чис­ла­ми 0,1, 0,01, 0,001.

    При­мер. Умно­жить 25,34 на 0,1.

    Вы­пол­ним за­пись де­ся­тич­ной дроби 0,1 в виде обык­но­вен­ной. Но умно­жить на  – то же самое, что раз­де­лить на 10. По­это­му необ­хо­ди­мо сдви­нуть за­пя­тую на 1 по­зи­цию влево:

    Ана­ло­гич­но умно­жить на 0,01 – это раз­де­лить на 100:

    При­мер. 5,235 раз­де­лить на 0,1.

    Ре­ше­ние дан­но­го при­ме­ра стро­ит­ся ана­ло­гич­ным об­ра­зом: 0,1 вы­ра­жа­ет­ся в виде обык­но­вен­ной дроби, а де­лить на  – это все равно, что умно­жить на 10:

    То есть чтобы по­де­лить на 0,1, нужно за­пя­тую сдви­нуть впра­во на одну по­зи­цию, что рав­но­силь­но умно­же­нию на 10.

    Правило умножения и деления на 0,1, 0,01…

    Умно­жить на 10 и раз­де­лить на 0,1 – это одно и то же. За­пя­тую нужно сдви­нуть впра­во на 1 по­зи­цию.

    Ана­ло­гич­но для 100, 1000 и так далее:

    Раз­де­лить на 10 и умно­жить на 0,1 – это одно и то же. За­пя­тую нужно сдви­нуть впра­во на 1 по­зи­цию:

    Ана­ло­гич­но для 100, 1000 и так далее:

    123,456:0,001 = 123 456

    Пра­ви­ло де­ле­ния де­ся­тич­ных дро­бей на на­ту­раль­ные числа.

    Че­ты­ре оди­на­ко­вых иг­руш­ки в сумме стоят 921 рубль 20 ко­пе­ек. Сколь­ко стоит одна иг­руш­ка (см. Рис. 1)?

    Рис. 1. Ил­лю­стра­ция к за­да­че

    Ре­ше­ние

    Для на­хож­де­ния сто­и­мо­сти одной иг­руш­ки необ­хо­ди­мо раз­де­лить дан­ную сумму на че­ты­ре. Пе­ре­ве­дём сумму в ко­пей­ки:

     

    Ответ: сто­и­мость одной иг­руш­ки 23030 ко­пе­ек, то есть 230 руб­лей 30 ко­пе­ек, или 230,3 рубля.

    Можно ре­шить дан­ную за­да­чу не пе­ре­во­дя рубли в ко­пей­ки, то есть раз­де­лить де­ся­тич­ную дробь на на­ту­раль­ное число: .

    Чтобы раз­де­лить де­ся­тич­ную дробь на на­ту­раль­ное число, нужно де­лить дробь на это число, как делят на­ту­раль­ные числа, и по­ста­вить в част­ном за­пя­тую тогда, когда за­кон­чит­ся де­ле­ние целой части.

    Делим в стол­бик так, как делят на­ту­раль­ные числа. После того как сно­сим цифру 2 (число де­ся­тых – пер­вая цифра после за­пя­той в за­пи­си де­ли­мо­го 921,20), в част­ном ста­вим за­пя­тую и про­дол­жа­ем де­ле­ние:

    Ответ: 230,3 рубля.

    Пример

    Делим в стол­бик так, как делят на­ту­раль­ные числа. После того как сно­сим цифру 6 (число де­ся­тых – цифра после за­пя­той в за­пи­си де­ли­мо­го 937,6), в част­ном ста­вим за­пя­тую и про­дол­жа­ем де­ле­ние:

    Ответ: .

    Пример

    Если де­ли­мое мень­ше де­ли­те­ля, то част­ное будет на­чи­нать­ся с нуля.

    1 на 19 не де­лит­ся, по­это­му в част­ном ста­вим ноль. Де­ле­ние целой части окон­че­но, в част­ном ста­вим за­пя­тую. Сно­сим 7. 17 на 19 не де­лит­ся, в част­ном пишем ноль. Сно­сим 6 и про­дол­жа­ем де­ле­ние:

    Ответ: .

    Деление десятичной дроби на 10,  100 и т.д.

    1) 

    Делим так, как делят на­ту­раль­ные числа. В част­ном по­ста­вим за­пя­тую сразу, как сне­сем 8 – первую цифру после за­пя­той в де­ли­мом 74,8. Про­дол­жа­ем де­ле­ние даль­ше. При вы­чи­та­нии  по­лу­ча­ем 8, но де­ле­ние не окон­че­но. Мы знаем, что в конце де­ся­тич­ной дроби можно при­пи­сы­вать нули – от этого зна­че­ние дроби не из­ме­нит­ся. При­пи­сы­ва­ем ноль и делим 80 на 10. По­лу­ча­ем 8 – де­ле­ние окон­че­но.

    Ответ: .

    2) 

    Ответ: .

    Чтобы раз­де­лить де­ся­тич­ную дробь на 10, 100, 1000 и т.д., надо пе­ре­не­сти за­пя­тую в этой дроби на столь­ко цифр влево, сколь­ко нулей стоит после еди­ни­цы в де­ли­те­ле.

    Пример

    Чтобы умно­жить дробь на целое число, нужно умно­жать, не об­ра­щая вни­ма­ния на за­пя­тую, затем в от­ве­те вер­нуть за­пя­тую на место, то есть от­де­лить ею столь­ко же цифр в дроб­ной части, сколь­ко было в ис­ход­ном числе.

    При­мер: 

    Вы­пол­няя умно­же­ние, не стоит об­ра­щать вни­ма­ния на за­пя­тую.

    Затем необ­хо­ди­мо по­ста­вить за­пя­тую так, чтобы в дроб­ной части по­лу­чи­лось, как и рань­ше, 3 цифры.

     

    Умножение десятичных дробей

    Для умно­же­ния двух де­ся­тич­ных дро­бей прин­цип аб­со­лют­но такой же.

    Пра­ви­ло

    Чтобы пе­ре­мно­жить две де­ся­тич­ные дроби, нужно их пе­ре­мно­жить, не об­ра­щая вни­ма­ния на за­пя­тые, затем в от­ве­те от­де­лить за­пя­той столь­ко цифр, сколь­ко их было у обоих чисел вме­сте.

     

    При­мер 1

    Сна­ча­ла необ­хо­ди­мо пе­ре­пи­сать каж­дую дробь в виде це­ло­го числа и вспо­мо­га­тель­но­го мно­жи­те­ля. Таким об­ра­зом, каж­дая дробь будет пред­став­ле­на в виде про­из­ве­де­ния.

    Затем нужно вы­пол­нить умно­же­ние целых чисел от­дель­но, вспо­мо­га­тель­ных мно­жи­те­лей от­дель­но. По­лу­чен­ный ре­зуль­тат нужно умно­жить на 0,001, то есть вы­пол­ня­ет­ся от­де­ле­ние за­пя­той дроб­ной части дли­ной в 3 цифры.

    При­мер 2

    Нужно вы­пол­нить умно­же­ние, не об­ра­щая вни­ма­ния на за­пя­тые. Ко­ли­че­ство цифр после за­пя­той – 3, по­это­му за­пя­тая ста­вит­ся, от­де­ляя 3 цифры. По­след­ний ноль можно убрать из за­пи­си.

    При­мер 3

    Вы­пол­ня­ет­ся умно­же­ние в стол­бик, при этом не об­ра­щая вни­ма­ния на за­пя­тые, но помня, что в конце надо будет также от­де­лить за­пя­той 4 цифры.

    При­мер 4

    В ходе урока мы уже вы­яс­ни­ли, что умно­жать де­ся­тич­ные дроби тех­ни­че­ски озна­ча­ет про­сто умно­жать целые числа. Далее в от­ве­те нужно от­де­лить за­пя­той зна­ков столь­ко, сколь­ко их было у всех чисел вме­сте.

    Ко­неч­но, это пра­ви­ло рас­про­стра­ня­ет­ся и на слу­чай несколь­ких мно­жи­те­лей:

     

    Деление десятичных дробей

    Си­ту­а­ция с де­ле­ни­ем де­ся­тич­ных дро­бей такая же: если уметь де­лить целые числа одно на дру­гое, то тогда по­лу­чит­ся и де­ся­тич­ную дробь де­лить на дру­гую де­ся­тич­ную дробь.

    При­мер

    Когда за­кан­чи­ва­ет­ся целое число, ко­то­рое надо раз­де­лить, то ста­вит­ся за­пя­тая и про­дол­жа­ет­ся вы­пол­не­ние вы­чис­ле­ний:

     

    При­мер

    Здесь си­ту­а­ция ровно такая же: как толь­ко кон­ча­ет­ся целая часть – ста­вит­ся за­пя­тая:

    То есть тех­ни­че­ски не важно, что де­лить на целое число – дробь или дру­гое целое число. Ал­го­ритм оди­на­ко­вый.

     

    Источник видео: https://www.youtube.com/watch?v=97n0NvNRPP0

    Источник конспекта: http://interneturok.ru/ru/school/matematika/5-klass/umnozhenie-i-delenie-desyatichnyh-drobey/umnozhenie-i-delenie-desyatichnyh-drobey?konspekt&chapter_id=1898

    Источник теста: http://testedu.ru/test/matematika/5-klass/dejstviya-s-desyatichnyimi-drobyami-2.html

    Математика для 5-х классов — Блок 2: Умножение и деление целых чисел

    Краткое описание агрегата

    В Блоке 2 учащиеся будут опираться на свою работу по многозначному умножению и делению с 4 класса, а также на свое понимание структуры десятичной системы в Блоке 1, чтобы завершить беглость с многозначным умножением и расширить умножение деление цифр для включения двузначных делителей.

    В 4-м классе ученики овладели навыками сложения и вычитания нескольких цифр (4.NBT.4), необходимый навык для вычисления сумм и разностей в стандартном алгоритме умножения и деления соответственно. Студенты также умножали целое число до четырех цифр на однозначное целое число, а также два двузначных числа (4.NBT.5). К концу 4 класса учащиеся могут вычислять эти продукты, используя стандартный алгоритм, но «неоднократно рассуждают о связи между математическими рисунками и письменными числовыми работами, помогая им увидеть алгоритмы умножения и деления как аббревиатуры или резюме своих рассуждения о количествах »(Progressions for CCSSM,« Number and Operation in Base Ten, K-5 », p.14). Студенты также находят частные и остатки целых чисел с дивидендами до четырех и однозначными делителями (4.NBT.6). Как и в случае с умножением, к концу 4-го класса ученики могут вычислять эти частные, используя стандартный алгоритм наряду с другими стратегиями и представлениями, чтобы алгоритмы были значимыми, а не зазубривались.

    Блок 2 5-го класса начинается с написания, оценки и интерпретации простых числовых выражений (5.OA.1, 5.OA.2). Это служит как для обзора основных фактов умножения и деления, которые поддерживают основной контент позже в модуле, так и как способ записи вычислений, которые будут становиться все более сложными по мере развития модуля.Затем учащиеся укрепляют стандартный алгоритм умножения с помощью вычислительных примеров из 4-го класса, прежде чем расширять его использование на все более и более крупные множители (5.NBT.5). Затем ученики следуют аналогичной прогрессии с делением, сначала вычисляя частные с учетом случаев из 4-го класса, используя различные стратегии, а затем распространяя эти методы на вычисления с использованием двузначных делителей. Однако обратите внимание, что, в отличие от умножения, свободное владение стандартным алгоритмом деления не ожидается до 6-го класса (6.NS.2). На протяжении всего модуля студенты «учатся использовать [] структуру [десятичных чисел] и свойства операций, чтобы сократить вычисление многозначного… продукта или частного до набора однозначных вычислений в различных десятичных единицах. (MP.7) (Развитие CCSSM, «Number and Operation in Base Ten, K-5», p. 4). Далее, «повторные рассуждения (MP.8), основанные на единообразии десятичной системы. система является частью этого процесса »(Progressions for CCSSM,« Number and Operation in Base Ten, K-5 », p.4).

    Позже в этом году студенты будут полагаться на свои знания в области целочисленных вычислений, чтобы выполнять их с десятичными знаками (5.NBT.7). Они также будут писать, оценивать и интерпретировать выражения с дробями и десятичными знаками в блоках 5 и 6 (5.OA.1, 5.OA.2). Эта работа «в 5 классе является прообразом [их] их более поздней работы с переменными выражениями» (6–8.EE) (OA Progression, стр. 32). Кроме того, студенты закрепляют беглость с помощью алгоритма деления и расширяют беглость всех четырех операций до десятичных дробей (6.EE.2–3). Они также распространяют эти вычисления на еще неизвестные части нашей системы счисления, такие как отрицательные числа (7.NS). Таким образом, к концу этого раздела студенты будут просто стесняться свободного владения всеми четырьмя операциями с целыми числами и будут готовы решать их с другими типами чисел, одновременно начав думать не только о вычислении числовых выражений, но и о понимании и интерпретации их. , намекая на новый мир математики — алгебру.

    Темп: 23 учебных дня (20 уроков, 2 гибких дня, 1 оценочный день)

    Чтобы узнать, как изменить темп на 2020-2021 учебный год в связи с закрытием школ, см. Наши рекомендации по корректировке объема и последовательности занятий для 5-го класса.

    Важные математические навыки для пятиклассников

    Хотите помочь своему пятикласснику освоить математику? Вот некоторые из навыков, которые ваш пятиклассник будет изучать в классе.

    Сложение, вычитание, умножение и деление

    Многозначные целые числа

    Быстро и точно умножайте многозначные целые числа. Разделите целые числа (до четырех цифр) на двузначные числа.

    Пример:

    Решить 4,824 ÷ 12 =?

    Объясните или проиллюстрируйте, как вы решили эту проблему.

    Совет: выделите практическое применение математики.

    По мере того, как математика, которую они изучают, становится более сложной и менее очевидно связанной с их повседневным опытом, у некоторых детей начинает развиваться математическая тревога. Важно, чтобы ваш ребенок занимался математикой и помогал ему понять, как в реальной жизни применяются концепции, которые ребенок изучает в школе. Составление бюджета на школьные принадлежности или на их ежемесячное пособие — это один из способов практиковать сложение и вычитание.Если вы попросите их помочь вам с приготовлением или выпечкой, это покажет им, как работают дроби. Помогать рассчитывать цены при покупке продуктов — тоже хорошая практика.

    Связанные

    Понимание разряда

    Расширьте понимание разряда: в многозначном числе цифра в одном месте представляет 1⁄10 того, что она представляет в месте слева от него, и в 10 раз больше как он изображен справа от него.

    Сравнение десятичных знаков

    Чтение, запись и сравнение десятичных знаков с разрядами тысячных, используя символы> (больше чем) и <(меньше чем).Например:

    • Прочтите это десятичное число: 23,002.
    • Запишите две и шестьдесят две тысячные в виде десятичного числа.
    • Какой знак подтверждает это утверждение: 5.389 _? _ 5.420
    • Исследователь измеряет количество бактерий, выросших на образцах неохлажденных продуктов. Ваш ребенок насчитывает 73,343 миллиона бактерий в образце A, 73,431 миллиона бактерий в образце B и 74,399 миллиона бактерий в образце C. Расположите образцы в порядке от наибольшего количества бактерий к наименьшему.Объясните или проиллюстрируйте, как вы приводите эти образцы в порядок.

    Связанные

    Десятичные доли до сотых

    Сложение, вычитание, умножение и деление десятичных долей до сотых.

    Совет: потренируйтесь в вычислениях с использованием десятичных знаков.

    Свяжите работу с десятичными знаками, которую ваш ребенок делает в классе, с реальным миром, поощряя их делать покупки по выгодным ценам. Попросите их разделить стоимость товаров, упакованных оптом, на количество отдельных товаров, чтобы определить стоимость каждого товара.Итак, сколько вы платите за рулон бумажного полотенца или за банку газировки при покупке оптом? Или попросите ребенка подсчитать, сколько вы сэкономите на каждом товаре, если цены со скидкой предполагают оптовые скидки.

    Что такое показатель степени

    Понять, что такое показатель степени. Например, «2» в 10² указывает, сколько раз нужно умножить число само на себя. 10² можно читать как «10 в степени 2», «10 в степени 2» или «10 в квадрате» и означает 10 x 10 или 100.10³ (или «10 в третьей степени» или «10 в кубе») означает 10 x 10 x 10, или 1000.

    Дроби

    Решение задач со словами

    Решение задач со словами, включающих сложение и вычитание дробей.

    Пример:

    Пятый класс собирает пазл из 600 деталей. Они начали вчера и собрали 100 частей — всего одну шестую (1⁄6) головоломки. Сегодня их собрано 400 штук. Какая часть головоломки завершена? Нарисуйте картинку И запишите математику, чтобы показать, как вы решили задачу.

    Совет: выделите практическое применение математики.

    По мере того, как математика, которую они изучают, становится более сложной и менее очевидно связанной с их повседневным опытом, у некоторых детей начинает развиваться математическая тревога. Важно, чтобы ваш ребенок занимался математикой и помогал ему понять, как в реальной жизни применяются концепции, которые он изучает в школе. Составление бюджета на школьные принадлежности или ежемесячное пособие — один из способов для нее практиковать сложение и вычитание.Если вы попросите ее помочь вам с приготовлением или выпечкой, это покажет ей, как работают дроби. Помогать рассчитывать цены при покупке продуктов — тоже хорошая практика.

    Нахождение общего знаменателя

    Решите задачи со словами, включающие сложение и вычитание дробей с разными знаменателями (нижние числа), преобразовывая их в дроби с одинаковым знаменателем, называемые общим знаменателем.

    Пример:

    Самая высокая девочка в пятом классе имеет рост 51 7⁄8 дюйма.Самый высокий мальчик в пятом классе имеет рост 49 сантиметров. Какая разница в их росте?

    После вечеринки остались две чашки лимонада. В одной миске 1⁄3 галлона. В другом — 1⁄2 галлона лимонада. Друг говорит, что не стоит пытаться объединить их в 1-галлонный контейнер, потому что лимонад вытечет наверх. Вы согласны? Почему или почему нет?

    Умножение дробей

    Решайте задачи со словами, включающие умножение дробей на другие дроби и умножение дробей на смешанные числа (целое число и дробь, например 11⁄4 или 21⁄2).

    Пример:

    • В оркестре средней школы 1⁄3 учащихся-музыкантов играют на струнных инструментах. Из учеников, играющих на струнных инструментах, 3⁄4 играют на скрипке. Какая часть оркестра играет на скрипке?
    • Утром во время экскурсии в яблоневый сад пятиклассники собрали 4⁄5 бушеля яблок. После обеда в полдень они собрали в 2,5 раза больше яблок. Уместятся ли все яблоки, собранные ими днем, в ящик на 2 бушеля? Откуда вы знаете?

    Совет: потренируйтесь использовать дроби.

    Помогите своему ребенку познакомиться с дробями, попросив его масштабировать рецепты для вашей семьи. Пусть они начнут с того, что уменьшат рецепт вдвое или вдвое. Когда они почувствуют себя комфортно, попросите их преобразовать его на 1 1/2, чтобы рецепт, который должен был накормить семью из четырех человек, работал на семью из шести человек.

    Единица деления дробей

    Разделите дроби единицы (дроби с 1 в числителе или верхним числом) на целые числа. Разделите целые числа на единичные дроби.

    Пример:

    Если три человека разделят ½ фунта шоколада поровну, сколько шоколада получит каждый? Объясните или проиллюстрируйте, как вы решили эту проблему.

    Умножение на дроби

    Помните, что умножение числа на дробь меньше 1 даст ответ меньше числа — например: 12 x ¾ = 9. Умножение числа на дробь больше 1 даст результат в ответе больше числа — например: 12 x 2 ½ = 30.

    Измерения и данные

    Преобразование единиц и дробей

    Преобразование единиц и долей единиц в одной системе измерения.

    Пример:

    Сколько минут составляет 1⁄5 часа? Объясните или проиллюстрируйте, как вы решили эту проблему.

    Проблемы многоступенчатого преобразования единиц измерения

    Решайте многоступенчатые задачи преобразования слов, используя преобразование стандартных единиц измерения разного размера.

    Пример:

    У меня 75 см ленты.Для выполнения проекта мне нужно в семь раз больше ленты. Сколько еще метров ленты мне нужно?

    Объясните или проиллюстрируйте, как вы решили эту проблему.

    Использование линейного графика

    Решайте проблемы, используя информацию (в единицах дроби), представленную на линейном графике.

    Геометрия

    Объем

    Под объемом понимается измерение пространства внутри трехмерной или твердой фигуры. Используйте формулы длина x ширина x высота или основание x высота , чтобы измерить объем трехмерного или твердого объекта с прямоугольными сторонами, например куба.Измеряйте объем для решения реальных проблем.

    Пример:

    Прямоугольный контейнер для мороженого имеет длину 8 дюймов и высоту 4 дюйма. Каков объем контейнера, выраженный в кубических дюймах?

    Советы, которые помогут вашему пятикласснику в уроке математики, можно найти на нашей странице с советами по математике для пятого класса.

    Ресурсы Parent Toolkit были разработаны NBC News Learn с помощью профильных экспертов и соответствуют Общим основным государственным стандартам.

    Бесплатные задания по математике для 5-го класса

    Вы здесь: Главная → Задания → 5 класс

    Это исчерпывающий набор бесплатных распечатываемых рабочих листов по математике для 5 класса, организованных по таким темам, как сложение, вычитание, алгебраическое мышление, разряд, умножение, деление, разложение на простые множители, десятичные дроби, дроби, измерения, координатная сетка и геометрия. Они генерируются случайным образом, их можно распечатать из вашего браузера и включать ключ ответа.Рабочие листы подходят для любой математической программы для пятого класса, но особенно хорошо подходят для программы IXL по математике для 5-го класса и их новых уроков внизу страницы.

    Рабочие листы генерируются случайным образом каждый раз, когда вы нажимаете на ссылки ниже. Вы также можете получить новый, другой, просто обновив страницу в своем браузере (нажмите F5).

    Вы можете распечатать их прямо из окна браузера, но сначала проверьте, как это выглядит в «Предварительном просмотре».Если рабочий лист не умещается на странице, отрегулируйте поля, верхний и нижний колонтитулы в настройках страницы вашего браузера. Другой вариант — настроить «масштаб» на 95% или 90% в предварительном просмотре печати. В некоторых браузерах и принтерах есть опция «Печатать по размеру», которая автоматически масштабирует рабочий лист по размеру области печати.

    Все рабочие листы содержат ключ ответа, расположенный на 2-й странице файла.


    Алгебра



    Математика для начальных классов Эдвард Заккаро

    Хорошая книга по решению проблем с очень разнообразными текстовыми задачами и стратегиями решения проблем.Включает главы по следующим темам: последовательности, решение проблем, деньги, проценты, алгебраическое мышление, отрицательные числа, логика, отношения, вероятность, измерения, дроби, деление. Вопросы в каждой главе разбиты на четыре уровня: легкий, несколько сложный, сложный и очень сложный.


    Сложение и вычитание в столбцах (числа друг под другом)


    Место и округление

    Пропускной счет

    • Пропуск на 20 000, начиная с 550 000
    • Пропуск на 50 000, начиная с 120 000
    • Пропуск на 100 000, начиная с 1 350 000
    • Пропуск на 100 000, начиная с 628 000
    • Пропуск на 300 000, начиная с 4 250 000
    • Пропуск на 500 000, начиная с 750 000

    Округление

    • Округлить до ближайшей десятки в пределах от 0 до 10 000
    • Округлить до ближайшей сотни в пределах от 0 до 1 000 000
    • Округлить до ближайшей тысячи, в пределах от 0 до 1 000 000

    • Смешанные задачи округления 1 — округление до ближайших десяти, сотен или тысяч
    • Смешанные задачи округления 2 — округление до ближайших десяти, сотен, тысяч или десяти тысяч
    • Смешанные задачи округления 3 — как указано выше, но округление до подчеркнутой цифры
    • Смешанные задачи округления 4 — округление до подчеркнутой цифры с округлением до ближайшего миллиона

    Умножение

    Умножение умственных способностей

    Длинное умножение (в столбцах)


    Отдел

    Психологическое отделение

    Длинное деление

    • 1-значный делитель, 4-значное делимое, без остатка
    • 1-значный делитель, 4-значное делимое, остаток

    • 2-значный делитель, 4-значное делимое, делитель между 11 и 35
    • 2-значный делитель, 4-значное делимое, без остатка — (делителем является любое двузначное число)
    • 2-значный делитель, 4-значное делимое, с остатком — (делителем является любое двузначное число)

    • Умножение уравнения (пропущенный множитель; решить в столбик)
    • Уравнения деления (отсутствует делимое или делитель; решить путем умножения или деления в столбик)

    Следующие четыре типа рабочих листов выходят за рамки Стандарт Common Core для пятого класса.

    Факторинг


    Дробное сложение и вычитание

    Как дроби / дробные части

    В отличие от дробей / дробных частей

    • Сложите или вычтите разные дроби — знаменатели 2, 3, 4, 5, 6, 8 и 10
    • Сложить или вычесть разные дроби — знаменатели 2-12
    • Задача: сложить или вычесть непохожие дроби — знаменатели 2-25
    • Задача: сложить или вычесть 3 в отличие от дробей — знаменатели 2, 3, 4, 5, 6, 8 и 10

    • Сложить или вычесть смешанные числа — знаменатели 2-12
    • Сложить или вычесть смешанные числа — знаменатели 2-25

    • Сложить или вычесть смешанное число и дробь или целое число — знаменатели 2-12
    • Сложить или вычесть смешанное число и дробь или целое число — знаменатели 2-25

    Умножение на дробь


    Фракционное деление

    Следующие типы рабочих листов выходят за рамки стандартов Common Core.


    Преобразование дробей в смешанные числа и наоборот


    Эквивалентные дроби и упрощенные дроби

    Записывать дроби как десятичные и наоборот

    В приведенных ниже таблицах ключ ответа не дает дроби в упрощенной форме. Например, 0,24 задается как 24/100, а не как 6/25. Если хотите, можете спросить студент упростить.



    Сложение десятичной дроби

    Ментальная математика

    От 0 до 1 десятичных цифр

    От 0 до 2 десятичных цифр

    Добавление колонки


    Десятичное вычитание

    Ментальная математика

    От 0 до 1 десятичных цифр

    От 0 до 2 десятичных цифр

    Задачи: ментальная математика

    Вычитание по столбцу

    Вызовы: алгебраическое мышление


    Десятичное умножение

    Ментальная математика

    • Умножение целого числа на десятичное — просто (одна десятичная цифра)
    • Умножить целое число на десятичное — сложнее (одна десятичная цифра)
    • Умножение целого числа на десятичное — пропущенный множитель (одна десятичная цифра)

    • Умножить целое число и десятичная дробь (1-2 десятичные цифры)
    • Умножить целое число и десятичное число — пропущенный множитель (1-2 десятичных знака)

    • Умножение целого числа на десятичное (1-3 десятичных знака)
    • Умножение целого числа на десятичное — пропущенный множитель (1-3 десятичных знака)

    • Умножение десятичных знаков на десятичные
    • Умножение десятичных знаков на десятичные — пропущенный множитель

    • Умножение десятичных знаков на десятичные или целые числа (смешанная практика)
    • Умножение десятичных дробей на десятичные или целые числа — пропущенный коэффициент (смешанная практика)

    • Умножить на 10 или 100 (1-2 десятичных знака)
    • Умножить на 10, 100 или 1000 (1-2 десятичных знака)
    • Умножить на 10, 100 или 1000 — пропущенный коэффициент (1-2 десятичные цифры)

    • Умножить на 10 или 100 (1-3 десятичных цифры)
    • Умножить на 10, 100 или 1000 (1-3 десятичных знака)
    • Умножить на 10, 100, 1000, 10000 или 100000 (1-3 десятичных знака)
    • Умножение десятичных знаков на 10, 100 или 1000 — отсутствующий множитель (1-3 десятичных знака)

    Умножить по столбцам


    Десятичное деление

    Ментальная математика

    Длинное деление


    Единицы измерения

    Обычная система

    • Преобразование между дюймы и футы — проще
    • Преобразование между дюймами и футами — сложнее

    • Преобразование между дюймы, футы и ярды — проще
    • Преобразование между дюймами, футами и ярдами — сложнее
    • Преобразование дюймов, футов и ярдов с десятичными знаками — используйте калькулятор

    • Преобразование миль, ярдов и футов 1 — с помощью калькулятора
    • Преобразование миль, ярдов и футов 2 — с помощью калькулятора

    • Преобразование между унциями и фунтами — проще
    • Преобразование между унциями и фунтами — сложнее

    • Преобразование между тоннами и фунтами — проще
    • Преобразование между тоннами и фунтами — сложнее

    • Преобразование между тоннами, фунтами и унциями с десятичными знаками — используйте калькулятор
    • Преобразование между чашками, пинтами и квартами
    • Преобразование между чашками, пинтами, квартами и галлонами
    • Преобразование между унциями, чашками и квартами

    • Все обычные единицы, кроме миль — смешанная практика
    • Все обычные единицы, кроме миль — смешанная практика — задача
    • Преобразование между различными обычными единицами с десятичными знаками — используйте калькулятор

    Метрическая система

    • Преобразование между мм, см и м — с использованием десятичных знаков
    • Преобразование между мм, см, м и км — с использованием десятичных знаков
    • Преобразование между мл и л и г и кг — с использованием десятичных знаков

    • Все метрические единицы, упомянутые выше — смешанная практика — с использованием десятичных знаков

    • Метрическая система: перевод единиц длины (мм, см, дм, м, плотина, гм, км)
    • Метрическая система: перевод единиц веса (мг, cg, dg, g, dag, hg, kg)
    • Метрическая система: преобразование единиц объема (мл, кл, дл, л, дал, гл, кл)
    • Метрическая система: преобразование единиц длины, веса и объема

    Сетка координат


    Геометрия



    Если вы хотите иметь больший контроль над такими параметрами, как количество задач, размер шрифта, интервал проблем или диапазон чисел, просто щелкните по этим ссылкам, чтобы самостоятельно использовать генераторы рабочих листов:


    Что такое длинное деление? [Определение, факты и пример]

    Игры с длинным разделением

    Разделить на 2-значные числа

    Разделить 4-значные числа на 2-значные числа, при этом от деления не остается остатка.Вы начнете с оценки частных.

    охватывает общий базовый учебный план 5.NBT.6Play NowРазделите на 2-значные числа с остатком

    Разделите 4-значные числа на 2-значные числа. Начните с оценки частных, которые пригодятся при делении на 2-значные числа.

    охватывает Common Core Curriculum 5.NBT.6Играть сейчасСмотреть все игры с разделами >>
    Учитесь с помощью полной программы обучения математике K-5

    Что такое длинное деление?

    В математике деление в столбик — это метод, используемый для деления больших чисел на группы или части.

    Деление в столбик помогает разбить проблему деления на последовательность более простых шагов. Как и во всех задачах деления, большое число, являющееся делимым, делится на другое число, которое называется делителем, чтобы получить результат, называемый частным, а иногда и остатком.

    Как вы делаете деление в столбик?

    Метод деления в столбик включает в себя основные математические операции.

    Для деления двух чисел этим методом рисуется таблица.Делитель пишется за пределами правых скобок, а делимое — внутри. Частное пишется над чертой сверху над дивидендом.

    Деление в столбик состоит из 5 шагов:

    D Разделить
    M Умножить
    S Вычесть
    B Обрушить
    R Повтор или остаток

    Вот пример деления в столбик с четким отображением каждого шага.

    Процесс начинается с деления или определения, сколько раз крайняя левая цифра делимого может делиться на делитель.

    Затем результат или ответ из шага 1, который становится первой цифрой частного, умножается на делитель и записывается под первой цифрой делимого.

    Вычитание производится по первой цифре делимого и записывается остаток.

    Следующая цифра делимого уменьшается, а затем процесс повторяется до тех пор, пока все цифры делимого не будут сброшены и не будет найден остаток.

    Как разделить десятичные дроби методом длинного деления?

    Деление в столбик можно также использовать для разделения десятичных чисел на равные группы. Он выполняет те же шаги, что и при делении в столбик, а именно: деление, умножение, вычитание, уменьшение и повторение или нахождение остатка.

    Вот пример деления в столбик с десятичными знаками.

    Интересные факты

    • 123454321 при делении на 11111 дает частное 11111 и остаток 0.

    Давайте споем!

    Если нужно разделить большие числа,

    нарисуйте таблицу для длинного деления сбоку.

    Напишите шаги, которые будут вашим руководством,

    D, M, S, B и R — Придерживайтесь долгого разделения!

    Давайте сделаем это!

    Вместо того, чтобы показывать видео для обучения полному делению или раздавать практические задания ученикам 4-го класса, приведите примеры из реальной жизни, когда они могут использовать метод длинного деления для деления.

    Скажем, при приготовлении кексов и печенья для продажи выпечки в школе вы можете попросить ребенка подсчитать количество партий, в которых можно приготовить печенье или кексы (исходя из количества форм на подносе), если общее количество печенья и кексы требуются. Вы также можете попросить их подсчитать общее количество необходимых картонных коробок, если в каждую коробку печенья помещается 15 печенья, а в картонную коробку для кексов — 6 кексов. Попросите их вычислить, используя метод длинного деления.

    Связанный математический словарь

    % PDF-1.4 % 448 0 объект > эндобдж xref 448 82 0000000016 00000 н. 0000002775 00000 н. 0000002922 00000 н. 0000003419 00000 п. 0000003751 00000 н. 0000004062 00000 н. 0000004260 00000 н. 0000004374 00000 н. 0000004486 00000 н. 0000004599 00000 н. 0000004708 00000 н. 0000005172 00000 п. 0000005199 00000 н. 0000005635 00000 н. 0000005662 00000 н. 0000006228 00000 п. 0000006624 00000 н. 0000006651 00000 п. 0000006678 00000 н. 0000007142 00000 н. 0000007282 00000 н. 0000007415 00000 н. 0000007550 00000 н. 0000007687 00000 н. 0000008233 00000 н. 0000008771 00000 п. 0000009330 00000 н. 0000009891 00000 н. 0000010435 00000 п. 0000010462 00000 п. 0000010856 00000 п. 0000010994 00000 п. 0000011520 00000 п. 0000011783 00000 п. 0000012286 00000 п. 0000012575 00000 п. 0000012846 00000 п. 0000014112 00000 п. 0000025731 00000 п. 0000025844 00000 п. 0000027008 00000 н. 0000027316 00000 п. 0000027431 00000 н. 0000030282 00000 п. 0000030600 00000 п. 0000030719 00000 п. 0000031960 00000 п. 0000032273 00000 п. 0000032400 00000 п. 0000034380 00000 п. 0000034704 00000 п. 0000034774 00000 п. 0000034854 00000 п. 0000038726 00000 п. 0000039000 00000 н. 0000039288 00000 п. 0000039358 00000 п. 0000039438 00000 п. 0000044340 00000 п. 0000044609 00000 п. 0000044923 00000 п. 0000044993 00000 п. 0000045073 00000 п. 0000069237 00000 п. 0000069502 00000 п. 0000069902 00000 н. 0000069972 00000 н. 0000070052 00000 п. 00000

    00000 п. 00000 00000 п. 00000 00000 п. 00000 00000 п. 00000

    00000 п. 0000091772 00000 п. 0000091987 00000 п. 0000092208 00000 п. 0000092447 00000 п. 0000112047 00000 н. 0000153155 00000 н. 0000214671 00000 н. 0000002590 00000 н. 0000001936 00000 н. трейлер ] / Назад 1142289 / XRefStm 2590 >> startxref 0 %% EOF 529 0 объект > поток hb«b`TA ؀, `׍ $ 009 | _9 GSXDqqE’KfZjM] Mg 斬` ᑮ M | JIS, * ygt2t, ptYoyD, & pIp8 ɯpDZ) DVmzFbSidv} | ¡v! -FgF6K9p13i5 ݘ ​​b> Lsw϶D # W7 * N4 sirY9yEʨoH: 9mE / o _? @ 3NJ

    120 задач по математике для учащихся 1–8 классов

    Вы сидите за партой, готовые вместе выполнить викторину, тест или задание по математике.Вопросы перетекают в документ до тех пор, пока вы не попадете в раздел с текстовыми проблемами.

    Помогла бы толчок творчества. Но этого не произошло.

    Независимо от того, являетесь ли вы учителем 3-го класса или учителем 8-го класса, готовящим учеников к старшей школе, воплощение математических концепций в примеры из реального мира, безусловно, может быть проблемой.

    Этот ресурс — ваш творческий заряд. Он предоставляет примеры и шаблоны математических задач на слова для 1-8 классов.

    Всего 120 примеров.Помогая вам разобраться в них, чтобы найти вопросы для ваших учеников, ресурс разделен на категории по следующим навыкам с некоторым перекрытием между темами:

    Список примеров дополнен советами по созданию увлекательных и сложных математических задач со словами.

    120 Математических задач со словами, классифицированных по навыкам

    Задачи со сложением

    Подходит для: 1-й, 2-й класс

    1. Добавление к 10: Ариэль играл в баскетбол.1 из ее выстрелов попал в обруч. 2 ее выстрела не попали в обруч. Сколько всего было выстрелов?

    2. Добавление к 20: У Адрианны есть 10 кусочков жевательной резинки, которыми она может поделиться со своими друзьями. На всех ее подруг не хватило жевательной резинки, поэтому она пошла в магазин за еще тремя кусочками жевательной резинки. Сколько кусочков жевательной резинки сейчас у Адрианны?

    3. Добавление к 100: У Адрианны есть 10 кусочков жевательной резинки, которыми она может поделиться со своими друзьями. На всех ее подруг не хватило жевательной резинки, поэтому она пошла в магазин и купила 70 кусочков клубничной жевательной резинки и 10 кусочков жевательной резинки.Сколько кусочков жевательной резинки сейчас у Адрианны?

    4. Добавление чуть больше 100: В ресторане 175 обычных стульев и 20 стульев для младенцев. Сколько всего стульев в ресторане?

    5. Добавляем к 1 000: Сколько печенья вы продали, если продали 320 шоколадных печений и 270 ванильных печений?

    6. Добавление и более 10 000: Обычно магазин товаров для хобби продает 10 576 торговых карточек в месяц. В июне в магазине товаров для хобби было продано на 15 498 карточек больше, чем обычно.В целом, сколько коллекционных карточек было продано в магазине для хобби в июне?

    7. Сложение 3 чисел: У Билли дома было 2 книги. Он пошел в библиотеку, чтобы достать еще 2 книги. Затем он купил 1 книгу. Сколько книг у Билли сейчас?

    8. Добавление трех чисел к 100 и более: Эшли купила большой мешок конфет. В сумке было 102 синих конфеты, 100 красных и 94 зеленых. Сколько всего было конфет?

    Задачи на вычитание слов

    Подходит для: 1-й класс, второй класс

    9.Вычитаем до 10: Всего в пиццерии было 3 пиццы. Покупатель купил 1 пиццу. Сколько пиццы осталось?

    10. Вычитая до 20: Ваша подруга сказала, что у нее 11 наклеек. Когда вы помогли ей убрать стол, у нее было всего 10 наклеек. Сколько наклеек не хватает?

    11. Вычитая до 100: У Адрианны есть 100 кусочков жевательной резинки, которыми она может поделиться с друзьями. Когда она пошла в парк, она разделила 10 кусочков клубничной жевательной резинки. Когда она вышла из парка, Адрианна поделилась еще 10 кусочками жевательной резинки.Сколько кусочков жевательной резинки сейчас у Адрианны?

    Зарегистрируйтесь сейчас

    12. Вычитание Немного больше 100: Ваша команда набрала 123 очка. В первом тайме было набрано 67 очков. Сколько было забито во втором тайме?

    13. Вычитаем до 1000: У Натана большая муравьиная ферма. Он решил продать несколько своих муравьев. Он начал с 965 муравьев. Продал 213. Сколько муравьев у него сейчас?

    14. Вычитая до 10 000 и более: Обычно магазин товаров для хобби продает 10 576 торговых карточек в месяц.В июле в магазине товаров для хобби было продано 20 777 коллекционных карточек. Сколько коллекционных карточек было продано в магазине в июле по сравнению с обычным месяцем?

    15. Вычитание 3 чисел: У Шарлин была упаковка из 35 мелков. 6 она отдала своей подруге Терезе. Она дала 3 своей подруге Мэнди. Сколько мелков осталось у Шарлин?

    16. Вычитание трех чисел из 100: Эшли купила большой мешок конфет, чтобы поделиться с друзьями. Всего конфет было 296 штук.Она подарила Мариссе 105 конфет. Еще она подарила Кайле 86 конфет. Сколько конфет осталось?

    Задачи умножения слов

    Подходит для: 2-й класс, 3-й класс

    17. Умножение однозначных целых чисел: Адрианне нужно разрезать форму с пирожными на части. Она нарезает на сковороду 6 ровных столбиков и 3 ровных ряда. Сколько у нее пирожных?

    18. Умножение 2-значных целых чисел: В кинотеатре 25 рядов сидений по 20 мест в каждом ряду.Сколько всего мест?

    19. Умножение целых чисел, заканчивающееся на 0: Компания по производству одежды предлагает 4 различных вида толстовок. Ежегодно компания производит 60 000 толстовок каждого вида. Сколько свитшотов компания производит каждый год?

    20. Умножение 3 целых чисел: Каменщик укладывает кирпичи в 2 ряда по 10 кирпичей в каждом ряду. Сверху каждого ряда находится стопка из 6 кирпичей. Сколько всего кирпичей?

    21.Умножение 4 целых чисел: Кэли зарабатывает 5 долларов в час, разнося газеты. Она доставляет газеты 3 дня в неделю по 4 часа за раз. Сколько денег заработает Кэли после доставки газет в течение 8 недель?

    Задачи с разделением слов

    Подходит для: 3-й класс, 4-й класс, 5-й класс

    22. Разделение однозначных целых чисел: Если у вас есть 4 леденца, равномерно разделенные на 2 пакета, сколько штук конфет в каждой сумке?

    23.Деление 2-значных целых чисел: Если у вас есть 80 билетов на ярмарку, и каждая поездка стоит 5 билетов, сколько поездок вы сможете совершить?

    24. Разделительные числа, оканчивающиеся на 0: У школы есть 20 000 долларов на покупку нового компьютерного оборудования. Если каждая единица оборудования стоит 50 долларов, сколько всего ее может купить школа?

    25. Разделение на 3 целых числа: Мелисса покупает 2 пачки теннисных мячей на общую сумму 12 долларов. Всего 6 теннисных мячей. Сколько стоит 1 упаковка теннисных мячей? Сколько стоит 1 теннисный мяч?

    26.Остальные переводчики: Итальянский ресторан получил партию из 86 котлет из телятины. Если на блюдо нужно 3 котлеты, сколько котлет останется в ресторане после приготовления как можно большего количества блюд?

    Задачи со смешанными операциями со словами

    Подходит для: 3-й, 4-й, 5-й класс

    27. Смешивание сложения и вычитания: В библиотеке 235 книг. В понедельник вывозят 123 книги. Во вторник возвращено 56 книг.Сколько сейчас книг?

    28. Смешивание, умножение и деление: Есть группа из 10 человек, которые заказывают пиццу. Если каждый человек получает 2 куска, а у каждой пиццы 4 куска, сколько пиццы им следует заказать?

    29. Смешивание, умножение, сложение и вычитание: У Ланы 2 пакета по 2 шарика в каждом. У Маркуса 2 сумки по 3 шарика в каждой. Сколько еще шариков у Маркуса?

    30. Подразделение смешивания, сложения и вычитания: У Ланы есть 3 мешка с одинаковым количеством шариков в них, всего 12 шариков.У Маркуса 3 сумки с таким же количеством шариков, всего 18 шариков. Сколько еще шариков у Маркуса в каждой сумке?

    Упорядочивание слов и задачи распознавания чисел

    Подходит для: 2-й класс, 3-й класс

    31. Подсчет для предварительного умножения: В вашем классе 2 классные доски. Если на каждую классную доску нужно 2 куска мела, сколько всего кусков вам нужно?

    32. Подсчет перед предварительным просмотром: В вашем классе 3 классные доски.На каждой доске по 2 мелка. Это означает, что всего есть 6 мелков. Если вы уберете по 1 мелу с каждой доски, сколько всего их будет?

    33. Составление чисел: Какое число 6 десятков и 10 единиц?

    34. Числа для угадывания: У меня 7 в разряде десятков. У меня четное число вместо единиц. Мне меньше 74. Какой я номер?

    35. В поисках заказа: В хоккейном матче Митчелл набрал больше очков, чем Уильям, но меньше очков, чем Остон.Кто набрал больше всего очков? Кто набрал меньше всего очков?

    Задачи со словами на дроби

    Подходит для: 3-й класс, 4-й класс, 5-й класс, 6-й класс

    36. Поиск фракций группы: Джулия пошла на Хэллоуин в 10 домов на своей улице. В 5 домах ей подарили плитку шоколада. В какой части домов на улице Джулии ей дали плитку шоколада?

    37. Поиск фракций единицы: Хизер рисует портрет своей лучшей подруги Лизы.Чтобы было легче, она делит портрет на 6 равных частей. Какая дробь представляет каждую часть портрета?

    38. Сложение дробей с одинаковыми знаменателями: Ной проходит ks километра до школы каждый день. Он также проходит ⅓ километра, чтобы вернуться домой после школы. Сколько всего километров он проходит?

    39. Вычитание дробей с одинаковыми знаменателями: На прошлой неделе Уитни подсчитала количество коробок сока, которые у нее были на школьные обеды. У нее было случая.На этой неделе осталось ⅕ случая. Сколько вина выпила Уитни?

    40. Сложение целых чисел и дробей с одинаковыми знаменателями: В обеденный перерыв в кафе-мороженом подавали 6 ложек шоколадного мороженого, 5 ложек ванили и 2 ложки клубники. Сколько всего шариков мороженого обслужили в салоне?

    41. Вычитание целых чисел и дробей с одинаковыми знаменателями: На вечеринке у Хайме было 5 ⅓ бутылок колы, чтобы ее друзья выпили.Она сама выпила бутылки. Ее друзья выпили 3 ⅓. Сколько бутылок колы осталось у Хайме?

    42. Сложение дробей с непохожими знаменателями: Кевин выполнил ½ задания в школе. Вернувшись в тот вечер домой, он выполнил ⅚ другого задания. Сколько заданий выполнил Кевин?

    43. Вычитание дробей с непохожими знаменателями: Собирая школьные обеды для своих детей, Пэтти использовала упаковки ветчины. Еще она использовала ½ упаковки индейки.Насколько больше ветчины, чем индейки, использовала Пэтти?

    44. Умножение дробей: Во время урока физкультуры в среду ученики пробежали километра. В четверг они пробежали ½ километра, как в среду. Сколько километров пробежали студенты в четверг? Запишите свой ответ дробью.

    45. Разделение на фракции: Производитель одежды использует флакона цветного красителя для изготовления одной пары брюк. Производитель вчера использовал бутылки. Сколько пар брюк изготовил производитель?

    46.Умножение дробей на целые числа: На этой неделе Марк выпил пакета молока. Фрэнк выпил в 7 раз больше молока, чем Марк. Сколько пакетов молока выпил Фрэнк? Запишите свой ответ дробью, целым или смешанным числом.

    Десятичные числа задач со словами

    Подходит для: 4-й класс, 5-й класс

    47. Добавление десятичных знаков: У вас есть 2,6 грамма йогурта в вашей миске, и вы добавляете еще одну ложку 1,3 грамма. Сколько всего йогурта у вас есть?

    48.Вычитание десятичных знаков: У Джеммы было 25,75 грамма глазури для приготовления торта. Она решила использовать только 15,5 грамма глазури. Сколько глазури осталось у Джеммы?

    49. Умножение десятичных дробей на целые числа: Маршалл проходит в общей сложности 0,9 км до школы и обратно каждый день. Сколько километров он пройдет через 4 дня?

    50. Разделение десятичных знаков на целые числа: Чтобы сделать Пизанскую башню из спагетти, миссис Робинсон купила 2 штуки.5 килограммов спагетти. Всего ее ученики смогли построить 10 наклонных башен. Сколько килограммов спагетти нужно для изготовления 1 падающей башни?

    51. Смешивание сложения и вычитания десятичных знаков: У Рокко в холодильнике 1,5 литра апельсиновой соды и 2,25 литра виноградной газировки. У Антонио 1,15 литра апельсиновой газировки и 0,62 литра виноградной газировки. Насколько больше газировки у Рокко, чем у Анджело?

    52. Смешивание умножения и деления десятичных знаков: 4 дня в неделю Лаура занимается боевыми искусствами на 1 ед.5 часов. Учитывая, что в неделе 7 дней, каково ее среднее время занятий в день каждую неделю?

    Сравнение и упорядочение словарных задач

    Подходит для: Детский сад, 1-й класс, 2-й класс

    53. Сравнение однозначных целых чисел: У вас 3 яблока, и у вашего друга 5 яблок. У кого больше?

    54. Сравнение 2-значных целых чисел: У вас 50 конфет, а у вашего друга 75 конфет. У кого больше?

    55.Сравнение различных переменных: На детской площадке есть 5 баскетбольных мячей. На детской площадке установлено 7 футбольных мячей. Есть еще баскетбольные мячи или футбольные мячи?

    56. Последовательность однозначных целых чисел: У Эрика 0 наклеек. Каждый день он получает еще 1 наклейку. Сколько дней до того, как он получит 3 наклейки?

    57. Пропуск по нечетным числам: Натали начала с 5. Она начала счет по пятеркам. Могла ли она сказать число 20?

    58. Пропуск по четным числам: Наташа начала с 0.Она пропустила счет до восьмерок. Могла ли она сказать число 36?

    59. Последовательность 2-значных чисел: Каждый месяц Джереми добавляет такое же количество карточек в свою коллекцию бейсбольных карточек. В январе у него было 36. В феврале 48. 60 марта. Сколько бейсбольных карточек будет у Джереми в апреле?

    Задачи со словом времени

    Подходит для: 1-й класс, 2-й класс

    66. Преобразование часов в минуты: Джереми помогал своей маме 1 час.Сколько минут он ей помогал?

    69. Добавление времени: Если вы просыпаетесь в 7:00 утра и вам нужно 1 час 30 минут, чтобы собраться и пойти в школу, в какое время вы придете в школу?

    70. Время вычитания: Если поезд отправляется в 14:00. и прибывает в 16:00, сколько времени пассажиры находились в поезде?

    71. Определение времени начала и окончания: Ребекка вышла из магазина своего отца, чтобы пойти домой в двадцать семь вечера.Через сорок минут она была дома. Во сколько она приехала домой?

    Задачи с деньгами

    Подходит для: 1-й, 2-й, 3-й, 4-й, 5-й класс

    60. Добавление денег: Томас и Мэтью копят деньги, чтобы вместе купить видеоигру . Томас сэкономил 30 долларов. Мэтью сэкономил 35 долларов. Сколько денег они накопили в общей сложности?

    61. Вычитание денег: Томас накопил 80 долларов. На свои деньги он покупает видеоигру.Видеоигра стоит 67 долларов. Сколько денег у него осталось?

    62. Умножение денег: Тим получает 5 долларов за доставку бумаги. Сколько у него будет денег после 3-х раздачи бумаги?

    63. Разделение денег: Роберт потратил 184,59 доллара на покупку трех хоккейных клюшек. Если бы каждая хоккейная клюшка имела одинаковую цену, сколько стоила бы 1 клюшка?

    64. Сложение денег с десятичными знаками: Вы пошли в магазин и купили жевательную резинку за 1,25 доллара и присоску за 0,50 доллара. Сколько было у вас всего?

    65.Вычитание денег с десятичными знаками: Вы пошли в магазин с 5,50 долларами. Вы купили жевательную резинку за 1,25 доллара, плитку шоколада за 1,15 доллара и присоску за 0,50 доллара. Сколько у тебя осталось денег?

    67. Применение пропорциональных отношений к деньгам: Якоб хочет пригласить 20 друзей на свой день рождения, что обойдется его родителям в 250 долларов. Если он вместо этого решит пригласить 15 друзей, сколько денег это будет стоить его родителям? Предположим, что отношение прямо пропорционально.

    68.Применение процентов к деньгам: Retta положила 100 долларов США на банковский счет, который приносит 20% годовых. Сколько процентов будет накоплено за 1 год? И если она не снимает деньги, сколько денег будет на счету через 1 год?

    Проблемы с физическими измерениями

    Подходит для: 1-й, 2-й, 3-й, 4-й класс

    72. Сравнение измерений: Линейка Кассандры имеет длину 22 сантиметра. Линейка апреля имеет длину 30 сантиметров.На сколько сантиметров длиннее линейка апреля?

    73. Измерения в контексте: Представьте школьный автобус. Какая единица измерения лучше всего описывает длину автобуса? Сантиметры, метры или километры?

    74. Добавление измерений: Папа Миши хочет сэкономить на бензине, поэтому он отслеживает, сколько он потребляет. В прошлом году папа Миши использовал 100 литров бензина. В этом году ее отец использовал 90 литров бензина. Сколько всего газа он использовал за два года?

    75.Вычитание измерений: Папа Миши хочет сэкономить на бензине, поэтому он отслеживает, сколько он потребляет. За последние два года папа Миши использовал 200 литров бензина. В этом году он использовал 100 литров газа. Сколько газа он использовал в прошлом году?

    76. Умножение объема и массы: Кира хочет убедиться, что у нее крепкие кости, поэтому она выпивает 2 литра молока каждую неделю. Сколько литров молока выпьет Кира через 3 недели?

    77. Разделение объема и массы: Лилиан занимается садоводством, поэтому она купила 1 килограмм земли.Она хочет равномерно распределить почву между двумя растениями. Сколько получит каждое растение?

    78. Преобразование массы: Ингер идет в продуктовый магазин и покупает 3 тыквы, каждая из которых весит 500 грамм. Сколько килограммов кабачков купила Ингер?

    79. Преобразование объема: У Шэда есть киоск для лимонада, и он продал 20 чашек лимонада. Каждая чашка была 500 миллилитров. Сколько литров всего продала Шад?

    80. Длина преобразования: Стейси и Мильда сравнивают свой рост.Рост Стейси 1,5 метра. Милда на 10 сантиметров выше Стейси. Какой рост у Милды в сантиметрах?

    81. Расстояние и направление: Автобус отправляется из школы, чтобы отвезти учащихся на экскурсию. Автобус едет на 10 километров на юг, 10 километров на запад, еще 5 километров на юг и 15 километров на север. В каком направлении должен ехать автобус, чтобы вернуться в школу? Сколько километров он должен пройти в этом направлении?

    Соотношение и процентное соотношение словарных задач

    Подходит для: 4-й класс, 5-й класс, 6-й класс

    82.В поисках недостающего числа: Соотношение трофеев Дженни и трофеев Мередит составляет 7: 4. У Дженни 28 трофеев. Сколько у Мередит?

    83. Поиск недостающих номеров: Соотношение трофеев Дженни и трофеев Мередит составляет 7: 4. Разница между числами — 12. Какие числа?

    84. Сравнительные показатели: В младшем школьном оркестре 10 саксофонистов и 20 трубачей. В старшем оркестре школы 18 саксофонистов и 29 трубачей.У какого оркестра более высокое соотношение трубачей и саксофонистов?

    85. Определение процентного соотношения: Мэри опросила учеников своей школы, чтобы определить их любимые виды спорта. 455 из 1200 студентов назвали хоккей своим любимым видом спорта. Какой процент студентов назвал хоккей своим любимым видом спорта?

    86. Определение процента изменения: Десять лет назад население Оквилла составляло 67 624 человека. Теперь он на 190% больше. Каково население Оквилля в настоящее время?

    87.Определение процентов чисел: На пункте проката коньков 60% из 120 коньков — для мальчиков. Если остальные коньки для девочек, сколько их?

    88. Расчет средних значений: В течение 4 недель Уильям вызвался помощником на занятиях по плаванию. Первую неделю он работал волонтером по 8 часов. Он работал волонтером 12 часов на второй неделе и еще 12 часов на третьей неделе. На четвертой неделе он работал волонтёром 9 часов. Сколько часов в среднем он работал волонтером в неделю?

    Вероятность и проблемы со связями данных

    Подходит для: 4-й, 5-й, 6-й, 7-й классы

    89.Понимание предпосылки вероятности: Джон хочет узнать любимое телешоу его класса, поэтому он опрашивает всех мальчиков. Будет ли выборка репрезентативной или необъективной?

    90. Понятие материальной вероятности: Грани на большом количестве кубиков помечены цифрами 1, 2, 3, 4, 5 и 6. Вы бросаете кубик 12 раз. Сколько раз вы должны ожидать, что вам выпадет 1?

    91. Изучение дополнительных событий: Цифры от 1 до 50 в шляпе. Если вероятность выпадения четного числа составляет 25/50, какова вероятность НЕ выпадать четное число? Выразите эту вероятность дробью.

    92. Исследование экспериментальной вероятности: В пиццерии недавно было продано 15 пицц. 5 из этих пицц были пепперони. Отвечая дробью, какова экспериментальная вероятность того, что следующая пицца будет пепперони?

    93. Введение в отношения данных: Маурита и Феличе проходят по 4 теста. Вот результаты 4 тестов Мауриты: 4, 4, 4, 4. Вот результаты 3 из 4 тестов Феличе: 3, 3, 3. Если среднее значение Мауриты по 4 тестам на 1 балл выше, чем у Феличе, каков результат? оценка 4-го теста Феличе?

    94.Представляем пропорциональные отношения: Магазин A продает 7 фунтов бананов за 7 долларов. Магазин B продает 3 фунта бананов по цене 6 долларов. В каком магазине выгоднее?

    95. Написание уравнений для пропорциональных отношений: Лайонел любит футбол, но у него проблемы с мотивацией к тренировкам. Итак, он стимулирует себя с помощью видеоигр. Существует пропорциональная зависимость между количеством упражнений, которые Лайонел выполняет, в x , и тем, сколько часов он играет в видеоигры, в x .Когда Лайонел выполняет 10 упражнений, он 30 минут играет в видеоигры. Напишите уравнение отношения между x и y .

    Геометрические задачи со словом

    Подходит для: 4-й, 5-й, 6-й, 7-й, 8-й классы

    96. Представляем Периметр: В театре 4 стула в ряд. Всего 5 рядов. Если использовать строки в качестве единицы измерения, каков периметр?

    97. Зона представления: В театре 4 стула в ряд.Всего 5 рядов. Сколько всего стульев?

    98. Введение Том: Аарон хочет знать, сколько конфет может вместить его контейнер. Контейнер имеет высоту 20 сантиметров, длину 10 сантиметров и ширину 10 сантиметров. Каков объем контейнера?

    99. Понимание 2D-форм: Кевин рисует фигуру с 4 равными сторонами. Какую форму он нарисовал?

    100. Обнаружение периметра 2D-форм: Митчелл написал свои домашние вопросы на листе квадратной бумаги.Каждая сторона бумаги по 8 сантиметров. Какой периметр?

    101. Определение площади 2D-форм: Одна торговая карточка имеет длину 9 см и ширину 6 см. Какая у него площадь?

    102. Что такое 3D-фигуры: Марта рисует фигуру с 6 квадратными гранями. Какую форму она нарисовала?

    103. Определение площади поверхности трехмерных фигур: Какова площадь поверхности куба шириной 2 см, высотой 2 см и длиной 2 см?

    104.Определение объема 3D-форм: Контейнер для конфет Аарона имеет высоту 20 сантиметров, длину 10 сантиметров и ширину 10 сантиметров. Контейнер Брюса имеет высоту 25 сантиметров, длину 9 сантиметров и ширину 9 сантиметров. Найдите объем каждого контейнера. В зависимости от объема, чей контейнер может вместить больше конфет?

    105. Определение прямоугольных треугольников: Треугольник имеет следующие длины сторон: 3 см, 4 см и 5 см. Этот треугольник прямоугольный?

    106.Определение равносторонних треугольников: Треугольник имеет следующие длины сторон: 4 см, 4 см и 4 см. Что это за треугольник?

    107. Определение равнобедренных треугольников: Треугольник имеет следующие длины сторон: 4 см, 5 см и 5 см. Что это за треугольник?

    108. Определение треугольников из чешуи: Треугольник имеет следующие длины сторон: 4 см, 5 см и 6 см. Что это за треугольник?

    109. Определение периметра треугольников: Луиджи построил палатку в форме равностороннего треугольника.Периметр 21 метр. Какова длина каждой стороны палатки?

    110. Определение площади треугольников: Какова площадь треугольника с основанием в 2 единицы и высотой 3 единицы?

    111. Применение теоремы Пифагора: Прямоугольный треугольник имеет длину одной стороны без гипотенузы 3 дюйма и длину гипотенузы 5 дюймов. Какова длина другой стороны без гипотенузы?

    112. Определение диаметра круга: Жасмин купила новый круглый рюкзак.Его площадь составляет 370 квадратных сантиметров. Какой диаметр у круглого рюкзака?

    113. Поиск области круга: Круглый щит Капитана Америки имеет диаметр 76,2 сантиметра. Какова площадь его щита?

    114. Поиск радиуса круга: Скайлар живет на ферме, где его отец держит круглый кукурузный лабиринт. Кукурузный лабиринт имеет диаметр 2 километра. Каков радиус лабиринта?

    Переменные задачи со словами

    Подходит для: 6-й, 7-й, 8-й класс

    115.Определение независимых и зависимых переменных: Виктория печет кексы для своего класса. Количество кексов, которые она готовит, зависит от того, сколько у нее одноклассников. Для этого уравнения м — это количество кексов, а c — количество одноклассников. Какая переменная является независимой, а какая зависимой?

    116. Написание переменных для сложения: В прошлом футбольном сезоне Триш забила г голов.Алекса забила на 4 гола больше, чем Триш. Напишите выражение, показывающее, сколько голов забила Алекса.

    117. Написание выражений переменных для вычитания: Элизабет ест здоровый, сбалансированный завтрак b раз в неделю. Мэдисон иногда пропускает завтрак. В целом Мэдисон съедает на 3 завтрака меньше в неделю, чем Элизабет. Напишите выражение, показывающее, сколько раз в неделю Мэдисон завтракает.

    118. Написание выражений переменных для умножения: В прошлом хоккейном сезоне Джек забил g голов.Патрик забил вдвое больше голов, чем Джек. Напишите выражение, показывающее, сколько голов забил Патрик.

    119. Написание выражений переменных для подразделения: У Аманды c плиток шоколада. Она хочет равномерно распределить плитки шоколада между 3 друзьями. Напишите выражение, показывающее, сколько плиток шоколада получит один из ее друзей.

    120. Решение уравнений с двумя переменными: Это уравнение показывает, как сумма, которую Лукас зарабатывает на внешкольной работе, зависит от того, сколько часов он работает: e = 12h .Переменная h показывает, сколько часов он работает. Переменная e показывает, сколько денег он зарабатывает. Сколько денег заработает Лукас, проработав 6 часов?

    Как легко составлять свои собственные математические задачи со словом и рабочие листы с задачами с текстом

    Вооружившись 120 примерами, чтобы зажечь идеи, создание собственных задач по математике со словом может заинтересовать ваших учеников и обеспечить согласованность с уроками. Do:

    • Ссылка на интересы студентов: Обрамляя свои текстовые проблемы интересами студентов, вы, вероятно, привлечете внимание.Например, если большая часть вашего класса любит американский футбол, задача измерения может включать расстояние броска известного квотербека.
    • Задайте тематические вопросы: Написание словесной задачи, отражающей текущие события или проблемы, может заинтересовать учащихся, давая им четкий, осязаемый способ применения своих знаний.
    • Включите имена учащихся: Назовите символы вопроса в честь учащихся — это простой способ сделать предмет понятным, помогая им справиться с проблемой.
    • Будьте ясны: Повторение ключевых слов определяет вопрос, помогая учащимся сосредоточиться на основной проблеме.

    Не нужно:

    • Тест на понимание прочитанного: Цветочный выбор слов и длинные предложения могут скрыть ключевые элементы вопроса. Вместо этого используйте краткие фразы и лексику на уровне своего класса.
    • Сосредоточьтесь на схожих интересах: Слишком много вопросов, связанных с интересами, такими как футбол или баскетбол, может оттолкнуть или оттолкнуть некоторых студентов.
    • Особые опасения: Включение ненужной информации вводит еще один элемент решения проблем, подавляющий многих учеников начальной школы.

    Ключ к дифференцированному обучению, словесные задачи, которые студенты могут связать и контекстуализировать, вызовут больший интерес, чем общие и абстрактные.

    Заключительные мысли о математических задачах со словами

    Вероятно, вы получите максимальную отдачу от этого ресурса, если будет использовать задачи в качестве шаблонов, слегка изменяя их, применяя приведенные выше советы. Таким образом, они будут более релевантными и интересными для ваших учеников.

    Тем не менее, наличие 120 задач по математике, соответствующих учебной программе, на кончиках ваших пальцев, должно помочь вам решать задачи по развитию навыков и давать задания, заставляющие задуматься.

    Результат?

    Более глубокое понимание того, как ваши ученики обрабатывают контент, и демонстрация понимания, помогая в вашем текущем подходе к обучению.

    Решение задач умножением и делением дробей и смешанных чисел

    Задачи на дробные слова с помощью интерактивных упражнений

    Пример 1. Если для изготовления платья требуется 5/6 ярдов ткани, то сколько ярдов потребуется для изготовления 8 платьев?

    Анализ: Чтобы решить эту проблему, мы преобразуем целое число в неправильную дробь.Затем мы умножим две дроби.

    Решение:

    Ответ: Для изготовления 8 платьев потребуется 6 и 2/3 ярда ткани.


    Пример 2: У Рене была коробка кексов, половину которой она отдала своему другу Хуану. Хуан отдал 3/4 своей доли своей подруге Елене. Какая дробная часть оригинальной коробки кексов досталась Елене?

    Анализ: Чтобы решить эту проблему, мы умножим эти две дроби.

    Решение:

    Ответ: Елене досталось 3/8 оригинальной коробки кексов.


    Пример 3: Класс математики Нины имеет длину 6 и 4/5 метра и ширину 1 и 3/8 метра. Какая площадь классной комнаты?

    Анализ

    : Чтобы решить эту проблему, мы умножим эти смешанные числа. Но сначала мы должны преобразовать каждое смешанное число в неправильную дробь.

    Решение:

    Ответ: Площадь аудитории 9 и 7/20 квадратных метров.


    Пример 4: Плитка шоколада имеет длину 3/4 дюйма. Если его разделить на части длиной 3/8 дюйма, то сколько это будет частей?

    Анализ: Чтобы решить эту задачу, мы разделим первую дробь на вторую.

    Решение:

    Ответ: 2 шт.


    Пример 5: У электрика есть кусок провода длиной 4 и 3/8 сантиметра. Она делит проволоку на кусочки длиной 1 и 2/3 сантиметра. Сколько у нее штук?

    Анализ: Чтобы решить эту проблему, мы разделим первое смешанное число на второе.

    Решение:

    Ответ: У электрика 2 и 5/8 куска провода.


    Пример 6: На складе 1 и 3/10 метров ленты. Если они разделят ленту на куски длиной 5/8 метров, то сколько кусков у них получится?

    Анализ: Чтобы решить эту проблему, мы разделим первое смешанное число на второе. Сначала мы преобразуем каждое смешанное число в неправильную дробь.

    Решение:

    Ответ: На складе будет 2 и 2/25 кусков ленты.


    Резюме: В этом уроке мы узнали, как решать задачи со словами, связанные с умножением и делением дробей и смешанных чисел.


    Упражнения

    Указания: вычтите смешанные числа в каждом упражнении ниже. Обязательно упростите свой результат, если необходимо. Щелкните один раз в ОКНО ОТВЕТА и введите свой ответ; затем нажмите ENTER. После того, как вы нажмете ENTER, в БЛОКЕ РЕЗУЛЬТАТОВ появится сообщение, указывающее, правильный или неправильный ваш ответ. Чтобы начать заново, нажмите ОЧИСТИТЬ.

    Примечание. Чтобы написать смешанное число четыре и две трети, введите в форму 4, пробел и затем 2/3.

    1. Одна партия печенья содержит 1 и 3/4 стакана растопленного шоколада. Сколько чашек растопленного шоколада нужно для изготовления 8 партий печенья?
    2. Тодд выпил 5/8 банки сока объемом 24 унции. Лайла выпила на треть меньше сока, чем Тодд. Сколько унций выпила Лила?
    3. Прямоугольный коврик имеет длину 3 и 2/3 фута и ширину 2 и 3/4 фута. Какова площадь коврика?
    4. У Джанет 5 и 3/4 сантиметра лакричника. Она делит лакрицу на кусочки длиной 1 и 7/8 сантиметра. Сколько кусочков солодки у нее будет?
    5.

    Добавить комментарий

    Ваш адрес email не будет опубликован. Обязательные поля помечены *